Учебные материалы по математике | Комплексные числа и их реализация тфкп | Matematiku5
Вузы по математике Готовые работы по математике Как писать работы по математике Примеры решения задач по математике Решить задачу по математике online

Комплексные числа и их реализация тфкп


Лекция 1

Комплексные числа и их реализация

Отыщи всему начало, и ты многое поймешь.

Козьма Прутков

Комплексные числа. Истоки и история. Необходимость расширения множества действительных чисел. Реализация комплексных чисел.

Вопрос к экзамену: Комплексные числа. Истоки и история.

При изучении новой теории необходимо учитывать конкретную историческую обстановку, в которой происходило ее зарождение и становление. Комплексные числа, функций от комплексных переменных и, вообще, комплексный анализ прошел довольно длинный путь эволюции от первой своей версии до той формы, в какой он пребывает в настоящее время. История комплексных чисел полна драматизма. Дж. Кардано впервые (1545) ввел мнимые числа, хотя считал их бесполезными, непригодными к употреблению. Р. Бомбелли (1572) впервые оценил пользу мнимых чисел при решении кубических уравнений. Числа вида при в 16-17 веках назывались мнимыми. Однако даже для многих крупных ученых 17 века сущность мнимых величин представлялась неясной, загадочной и мистической. И. Ньютон не включал мнимые величины в понятие числа. Систематическое использование мнимых величин в задаче о степени начал М. Муавр (1707,1724) и Р. Котес (1722). через обозначил Л. Эйлер (1777). К. Гаусс (1799) доказал, что множество комплексных чисел алгебрайчески замкнуто. Термин комплексное число ввел Л. Карно (1803). Геометрическую интепретацию комплексных чисел дали К. Вессель (1799), Ж. Арган (1803). В 1851 году Б. Риман защитил докторскую диссертацию на тему: «Основы общей теории функций одной комплексной переменной». Эту дату можно считать официальным рождением новой дисциплины ”Теория функции комплексного переменного” или кратко ТФКП. В Советском Союзе первую докторскую диссертацию по ТФКП защитил А. И.Маркушевич, который школу закончил в Семипалатинске. В Казахстане отдельные разделы ТФКП получили развитие в работах академиков Н. К.Блиева и М. Отелбаева. Первые лекции по ТФКП на казахском языке начал читать Б. Тулегенов.

Современную математику трудно представить без комплексных чисел. Теория функций комплексного переменного – одна из красивейших ветвей математики. Цель читаемого курса показать красоту и мощь данного раздела математики. Надо отметить, что в этом курсе изучаются только функций заданные на комплексной плоскости. За его пределами остались такие бурно развивающиеся разделы комплексного анализа, как теория функций на римановых поверхностях, теория функций многих комплексных переменных.

Вопрос к экзамену: Необходимость расширения множества действительных чисел.

Натуральные числа необходимы для счета небольших количеств. Уравнение во множестве натуральных чисел не имеет решения, поэтому возникла проблема расширения множества натуральных чисел. В результате возникло его расширение — это множество целых чисел. Уравнение во множестве целых чисел не имеет решения, поэтому возникла проблема расширения множества целых чисел. В результате возникло его расширение — это множество рациональных чисел. Уравнение во множестве рациональных чисел не имеет решения, поэтому возникла проблема расширения множества рациональных чисел. В результате возникло его расширение — это множество действительных чисел [1]. Уравнение во множестве действительных чисел не имеет решения, поэтому возникла проблема расширения множества действительных чисел.[2] В результате возникло его расширение — это множество комплексных чисел. Отметим, что при этом не все свойства действительных чисел наследуются комплексными числами. Генетическими оказались следующие свойства действительных чисел:

    Коммутативность сложения и умножения; Дистрибутивность сложения относительно умножения; Ассоциативность; Существование единичного и нулевого элементов; Существование обратных и противоположных элементов.

В то же время множество комплексных чисел неупорядочено, хотя произвольные два действительных числа подчинены отношению порядка.

Вопрос к экзамену: Реализация комплексных чисел.

Напомним несколько различных реализации множества действительных чисел.

1.  Действительное число – это десятичная (конечная или бесконечная ) дробь.

2.  Действительное число – это точка на числовой оси.

3.  Действительное число – это точка на проколотой окружности.

Перечисленные множества изоморфны между собой в том смысле, что между каждой парой множеств существует взаимно-однозначное соответствие, которое сохраняет операции сложения и умножения. В математике изучаются не отдельные объекты (множества), а целые классы изоморфных между собой объектов. К примеру, следующее множество

также изоморфно множеству действительных чисел, то есть элементы множества также представляют собой действительные числа.

Цель: решить уравнение во множестве действительных чисел, то есть решить указанное уравнение хотя бы в одной из реализации действительных чисел.

Теорема 1. Матрица удовлетворяет матричному уравнению .

Доказательство заключается в непосредственной проверке матричного соотношения

Введем множество специальных матриц

Теорема 2.Множество есть расширение множества и матрица .

Доказательство. Если , то матрица вида принадлежит множеству .Если , то матрица вида совпадает с матрицей .

Таким образом, построено расширение множества действительных чисел, в котором уравнение разрешимо. Отметим, что подобных расширении много.

Упражнение 1. для вдумчивого читателя. Построить расширения, состоящие из квадратных матриц размерности или .

Упражнение 2. Проверить, что произведение двух матриц вида снова матрица такого же вида.

Упражнение 3. Проверить, что произведение двух матриц вида коммутативно.

Замечание для эмоциональных читательниц. Результат упражнения 3 ошеломляющий, поскольку произведение двух матриц не всегда коммутативно.

Упражнение 4. Вычислить определитель матрицы . Когда такая матрица вырожденна?

Вопрос для творческих натур. Если бы вырожденных матриц в было бы много, то к каким неприятным следствиям это привело бы?

Лекция 2

Различные формы записи комплексных чисел

Математика изучает не предметы, а лишь

отношения между ними; поэтому для них совершенно

безразлично, будут ли одни предметы замещены

другими, лишь бы только не менялись их отношения.

Анри Пуанкаре

1.  Алгебраическая форма записи комплексных чисел.

2.  Тригонометрическая форма записи комплексных чисел.

3.  Показательная форма записи комплексных чисел.

4.  Геометрическая интерпретация операции над комплексными числами.

Вопрос к экзамену: Алгебраическая форма записи комплексных чисел.

Согласно предыдущей лекции комплексное число – это матрица специального вида . Для дальнейших рассуждений удобно преобразовать эту матрицу в виде линейной комбинации двух фиксированных матриц.

Единичная матрица принадлежит множеству , поэтому сответствует действительному числу и часто будем называть ее просто единицей. Вторая матрица совпадает с матрицей , которая введена в теореме 1 и в дальнейшем будет называться мнимой единицей.. Итак, справедливо

Утверждение 1. Произвольное комплексное число представляет собой линейную комбинацию единицы и мнимой единицы, то есть комплексное число имеет вид или просто . Надо помнить, что коэффициенты линейной комбинации действительные числа. Коэффициент при единице называется реальной частью комплексного числа, а коэффициент при мнимой единице – мнимой частью комплексного числа.

Таким образом, получена алгебраическая форма записи комплексного числа.

Сложение и вычитание комплексных чисел проводится покомпонентно, то есть

    реальная часть суммы равна сумме реальных частей слагаемых, мнимая часть суммы равна разности мнимых частей слагаемых.

Умножение комплексных чисел в алгебраической записи вычисляется гораздо сложнее:

    ,

Приведенное правило умножения трудно запомнить. Лучше формально раскрыть скобки: и учесть, .

Вопрос к экзамену: Тригонометрическая форма записи комплексных чисел.

Согласно предыдущей лекции комплексное число – это матрица специального вида . Показана алгебраическая запись комплексного числа , то есть каждое комплексное число имеет две характеристики: реальная и мнимая части. Иначе говорят, что комплексное число однозначно определяется парой действительных чисел . Известно, что пара определяет вектор на плоскости.

рис. 1 Комплексное число в полярной системе координат.

 

 

Комплексное число в полярной системе координат записывается в виде , где — длина вектора , — угол соответствующий направлению вектора . Подробности показаны на рисунке 1. Величину называют модулем, угол аргументом комплексного числа и обозначают и соответственно. К примеру, . Представление называют тригонометрической формой записи комплексного числа.

Вопрос к экзамену: Показательная форма записи комплексных чисел.

Вспомним различные представления комплексного числа

    в виде матрицы частного вида, в виде линейной комбинации единицы и мнимой единицы, в полярной системе координат.

В настоящем пункте покажем еще одно представление для комплексных чисел, для вывода которого необходимо

Упражнение 5. Степень мнимой единицы может принимать только четыре значения

Доказательство. Сначала непосредственным умножением матриц проверим соотношения

. Затем для запишем цепочку очевидных равенств . Точно такие же выкладки справедливы и для других значений .

Упражнение 6. Для справедливо разложение в ряд Тейлора, использующее степени мнимой единицы

Доказательство. Сначала запишем стандартное разложение Тейлора . . Согласно упражнению 5 четные степени мнимой единицы поочередно принимают два значения: +1 и -1. В результате получим тезис упражнения 6.

Аналогично доказательству упражнения 6 доказывается следующее

Упражнение 7. Произведение раскладывается в ряд Тейлора

в котором отсутствует стандартное чередование знаков при разложении тригонометрических функции в ряд Тейлора.

Тривиальным следствием предыдущих упражнений является

Теорема 3 Экспоненциальная функция с чисто мнимым аргументом представляет линейную комбинацию тригонометрических функций, то есть справедливо равенство при всех действительных

Доказательство. Согласно упражнению 6 имеем соотношение

Из упражнения 7 следует равенство . Остается сложить правые части выписанных соотношений и вспомнить для экспоненты разложение в ряд Тейлора.

В качестве следствия из теоремы 3 получаем экспоненциальное представление комплексного числа , где — называется модулем комплексного числа, — называется аргументом комплексного числа.

Вопрос к экзамену: Геометрическая интерпретация операции над комплексными числами.

Таким образом, для комплексных чисел получены несколько представлений: матричное, алгебраическое, тригонометрическое и показательное. Было показано, что матричное представление обосновывает существование комплексных чисел. Покажем, что алгебраическое представление наглядно при геометрическом толковании операции сложения комплексных чисел. Экспоненциальное представление полезно при умножении и вычислении корня некоторой степени из комплексного числа. Формулу Муавра лучше всего записать, используя тригонометрическое представление.

Геометрическая интерпретация сложения.

Сложение комплексных чисел производится по правилу треугольника, как это происходит с векторами. Сложение удобнее производить в алгебраической записи:

Геометрическая интерпретация умножения.

Умножение удобно осуществлять, используя экспоненциальную форму записи комплексных чисел.

Геометрически сложение аргументов комплексных чисел означает поворот против часовой стрелки числа с большим аргументом на угол равный меньшему аргументу. На рисунке повернут на угол , так как .

Упражнение 8. Дать геометрическую интерпретацию делению.

Геометрическое изображение корней натуральных степеней из комплексных чисел. Поскольку , то или . Следовательно, для нахождения аргумента одного из корней аргумент надо разделить на . С другой стороны,

где — первый корень. Отсюда следует, что последующие корни отличаются от первого корня только значением аргумента. Иначе говоря, последующие корни получаются поворотом первого корня на угол . Таких поворотов на плоскости может быть ровно . Таким образом, все корни (их ровно ) лежат на одной окружности и делят эту окружность на равные дуги. На рисунке приведены корни четвертой степени из мнимой единицы.

Алгоритм определения корней — той степени из комплексного числа.

аргумент числа делим на и на плоскости рисуем луч соответствующий полученному углу. на луче отложим длину равную корню из модуля исходного числа. через полученную точку проводим окружность с центром в начале координат.

поделим окружность на равных частей так, чтобы одна из точек деления совпала с отмеченной точкой на луче.

Лекция 3

Топология комплексной плоскости

Математики, как французы: все что вы

им говорите, они переводят на свой язык, и это

тотчас же становится чем-то совершенно иным.

И. В.Гете.

Топология комплексной плоскости. Сходимость степенных рядов на комплексной плоскости.

Вопрос к экзамену: Топология комплексной плоскости.

Каждое комплексное число имеет модуль. Замечательно, что модуль разности двух чисел обладает следующими свойствами:

    неотрицательность, то есть при любых ; тогда и только тогда, когда ; симметричность, то есть при любых ; неравенство треугольника, то есть при любых .

Последнее свойство означает, что модуль разности между двумя числами достигает минимума на всевозможных расстояниях между этими числами. Иначе говоря, выполняется экстремальное соотношение

Поскольку модуль разности обладает указанным экстремальным свойством, то ее называют расстоянием между этими числами, а отрезок на котором достигается минимум — геодезической линией. Как только введено расстояние между элементами, то можно ввести понятие сходимости, предела, непрерывности и так далее. Причем определения данные в курсе математического анализа дословно повторяются. Хотя по смыслу иногда могут отличаться от того, что было в действительном анализе.

Открытый шар на плоскости , где — его центр, а — радиус. представляет окрестность точки . Внутренняя точка множества, если точка принадлежит множеству вместе с некоторой своей окрестностью. Открытое множество состоит только из внутренних точек. Любое открытое множество представляет объединение некоторых открытых шаров с центрами в точках множества. Замкнутое множество – это дополнение открытого множества. Любое замкнутое множество представляет пересечение дополнении открытых шаров с центрами во внешности множества. Топология – это множество всех открытых множеств. По определению пустое множество считаем открытым. Комплексное число назовем пределом последовательности комплексных чисел , если для каждого открытого шара с центром в точке вся последовательность, кроме, быть может, конечного числа ее элементов, принадлежит этому шару. Тогда говорят, что последовательность сходящаяся. Комплексное число предел функции в точке , которая является предельной точкой области определения функции, если для каждого открытого шара с центром в точке существует проколотая окрестность точки такая, что возможные ее -образы содержатся в наперед выбранном открытом шаре с центром в . Когда предел функции в точке совпадает со значением функции в этой точке, то функция непрерывна в соответствующей точке.

Определения известные из курса математического анализа можно продолжить. Лучше приведу одну диаграмму, по которой результаты математического анализа можно перенести и на комплексный анализ.

На диаграмме видно, что существует взаимно-однозначное соответствие не только между комплексной плоскостью и множеством , но и между комплекснозначными функциями и парой действительнозначных функций от двух переменных. Поэтому исследование сходимости, непрерывности функции комплексного переменного можно заменить эквивалентной задачей изучения пары действительных функций от двух переменных. К примеру, функция непрерывна на комплексной плоскости, так как непрерывна пара функций .

В заключении приведу один критерии непрерывности, который почему-то отсутствует в университетских учебниках. Хотя критерии используется в экономике, точнее в теории потребления при доказательстве непрерывности функции полезности.

Теорема 4. Функция непрерывна на области определения тогда и только тогда, когда прообраз каждого замкнутого относительно множества значении замкнут относительно области определения.

Вопрос к экзамену: Сходимость степенных рядов на комплексной плоскости.

Для полноты изложения подробнее изучим сходимость степенных рядов. Рассмотрим степенной ряд . Вспомним, что степенные ряды в действительном анализе обладали рядом замечательных свойств:

    имеет симметрический интервал сходимости с центром в точке и вне этого интервала расходится; формальное дифференцирование не уменьшает и не увеличивает радиуса интервала сходимости; внутри интервала сходимости степенной ряд сходится равномерно.

Указанные свойства сохраняют силу и для степенных рядов на комплексной плоскости, если интервал сходимости заменить на круг сходимости. Больше того сохраняются и доказательства для степенных рядов из курса математического анализа, если вместо абсолютной величины понимать модуль комплексного числа.

Упражнение 9. Если степенной ряд сходится в точке , то он равномерно сходится при всех , для которых выполняется неравенство .

Для доказательства достаточно записать ряд в эквивалентной форме и вспомнить, что последовательность ограничена, модуль отношения меньше единицы.

Отсюда как следствие следует существование круга сходимости. Отметим, что радиус сходимости можно находить по формуле Коши-Адамара. Позднее будет доказано, что для радиуса сходимости справедлива формула

Утверждение 2. Радиус сходимости равен расстоянию от центра до ближайшей особой точки суммы ряда. Иначе говоря, на границе круга сходимости обязательно наличие особой суммы ряда. Напомним, что в особой точке сумма ряда неограниченна или нарушение голоморфности происходит по какой-то другой причине[3].

Подобное утверждение может не выполняться для степенных рядов из действительных чисел, если рассматривать сходимость на оси. К примеру, ряд имеет радиус сходимости равный единице, причем ряд сходится на границе интервала . Непонятно, что стало преградой для сходимости ряда вне интервала . В то же время степенной ряд имеет сумму, которая имеет мнимую единицу в качестве особой точки. За счет ветвления в указанной точке радиус сходимости не может быть больше единицы.

Лекция 4

Голоморфные функции

Я посетил также математическую школу, где учитель преподает по такому методу, какой едва ли возможно представить себе у нас в Европе. Каждая теорема с доказательством тщательно переписывается на тоненькой облатке чернилами, составленными из микстуры против головной боли. Ученик глотает облатку натощак и в течение следующих трех дней не ест ничего, кроме хлеба и воды. Когда облатка переваривается, микстура поднимается в его мозг, принося с собой туда же теорему.

Джонатан Свифт, «Путешествие Гулливера»

1.  Голоморфные функции.

2.  Необходимые условия голоморфности функции в точке.

3.  Достаточные условия голоморфности функции в точке.

Вопрос к экзамену: Голоморфные функции.

Дифференцируемые функций в смысле комплексного анализа существенно отличаются от дифференцируемых в смысле действительного анализа. К примеру, непрерывные всюду функций, но нигде не имеющие производных в действительном анализе, строятся довольно изощренным образом. В то же время в комплексном анализе такие функций встречаются гораздо чаще. Несмотря на это словесные определения дифференцируемости в комплексном и действительном анализах совпадают.

В дальнейшем области определении функций считаются открытыми множествами, то есть мы изучаем свойства функций только во внутренних точках. Как исследуются граничные значения функции комплексного переменного можно познакомиться в продвинутых лекциях по ТФКП. Производной функции в точке называется предел . Напомним, что внутренняя точка области определения функции . Производная обозначается стандартным образом через .Для сравнения приведем параллельно определения действительного анализа.

означает следующее:

при

Последнее включение геометрически при достаточно малом означает:

Внутри круга может стремиться к по разным направлениям, причем предел (производная) не должен зависеть от направления стремления. Иначе говоря, пределы по всем направления существуют и равны между собой.

означает следующее:

при

Последнее включение геометрически при достаточно малом означает:

 

 

 

Внутри интервала может стремиться к только по двум направлениям ( слева и справа ), причем предел слева должен совпадать с пределом справа.

Таким образом, в случае предела на комплексной плоскости требований о совпадений пределов по разным направлениям гораздо больше, чем на оси. Поэтому естественно, что существование производной в смысле комплексного анализа гораздо редкое явление, чем аналогичный факт на оси. Действительно, даже самая на первый взгляд простая функция не имеет производной в смысле комплексного анализа. Чтобы в этом убедиться, вычислим последовательно

приращение приращение предел отношения , когда стремиться к по горизонтали. В этом случае , поэтому предел равен единице. предел отношения , когда стремиться к по вертикали. В этом случае , поэтому предел равен минус единице.

Если бы производная существовала, то пределы получались бы одинаковыми. Но в нашем случае пределы получились разными.

У некоторых возникло чувство неудовлетворенности, поскольку простенькая функция нигде не имеет производную. Хотя всюду непрерывна, так как таковыми являются ее реальная и мнимая части. Подобные же ощущения возникают и у корифеев математики. Ш. Эрмит писал в письме к Стильтьесу «С омерзением и ужасом я отворачиваюсь от этой зловредной язвы – непрерывных функций, нигде не имеющих производных». Для относительного успокоения приведу пример дифференцируемой функции.

Утверждение 3. Сумма степенного ряда имеет производные во всех точках круга сходимости, то есть дифференцируемыми будут, по крайней мере, те функций, которые раскладываются хотя бы локально в ряды Тейлора.

Доказательство. Пусть при степенной ряд сходится к . Не умаляя общности, можно считать . Если , то последовательность ограничена при , скажем, числом . Считая, что запишем разность в виде степенного ряда

,

где и . Выражение в фигурной скобке оценивается так

Отсюда получаем оценку модуля разности

Когда стремится к нулю, правая часть последнего неравенства также стремится к нулю. Следовательно, имеем предельное равенство , из которого следует существование производной суммы степенного ряда.

В заключении отметим, что голоморфной в точке функцией называют функцию, имеющую в точке производную в смысле комплексного анализа. Таким образом, не является голоморфной ни в одной точке плоскости, а сумма степенного ряда в круге сходимости представляет собой голоморфную функцию.

Вопрос к экзамену: Необходимые условия голоморфности функции в точке.

Как же отличать голоморфные в точке функций от неголоморфных? Пусть функция голоморфна во внутренней точке области определения . Это означает, что пределы

не зависят от угла . Сравним указанные пределы при и .

где .Отсюда следует, что частные производные реальной и мнимой частей исходной функции зависимы между собой, то есть

(K. – R.)

Эти условия в некоторых книгах называют условиями Коши Римана.[4] Если хотя одно из приведенных условий нарушается, то функция не будет голоморфной в точке. Таким образом, условия Коши-Римана являются необходимыми условиями голоморфности функции в точке. На самом деле, необходимых условий бесконечное число, так как принимает много значений. Удивительным фактом является то, что условия Коши-Римана «почти» достаточны. То есть оказывается достаточным записать необходимые условия для двух углов , отличающихся друг от друга .

Вопрос к экзамену: Достаточные условия голоморфности функции в точке.

Соотношения Коши-Римана являются необходимыми условиями голоморфности функции в точке. Однако они не достаточны. Действительно, для функции в точке условия Коши-Римана выполняются, хотя функция не дифференцируема в этой точке.[5] Чтобы убедиться в этом надо рассмотреть отношение , в котором положим . В результате предел зависит от , что противоречит существованию производной. В приведенном примере частные производные реальной части функции не являются непрерывными в точке . При устранении подобного дефекта частных производных реальной и мнимой частей условия Коши-Римана обеспечивают голоморфность функции.

Теорема 5. Предположим, что все четыре частные производные первого порядка реальной и мнимой частей существуют в окрестности точки . Тогда соотношения Коши-Римана составляют необходимое и достаточное условие голоморфности исходной функции в точке .

Из условии теоремы следует, что реальная часть как функция двух переменных представляет дифференцируемую в точке функцию. Следовательно, приращение реальной части в окрестности имеет вид . Подобным же образом получаем соотношение . Поскольку . Отсюда, учитывая соотношения Коши-Римана, получаем равенство

Отсюда следует существование производной функции в точке . Достаточность доказана. Необходимость доказывалась раньше.

Лекция 5

Различные интерпретации голоморфных функций

Не облекайте истину покровом лжи

И не старайтесь утаить ее,

Когда вы (суть) ее уже постигли.

Коран. 2 сура. 42 аят.

Геометрическая интерпретация модуля и аргумента производной голоморфной функции. Гидродинамическая интерпретация голоморфной функции.

Вопрос к экзамену: Геометрическая интерпретация модуля и аргумента производной голоморфной функции.

Сначала изучим вопрос: « Чем приращение голоморфной функции отличается от приращения функции, не обладающей свойством голоморфности?» Если не предполагать голоморфности, то для приращения имеем соотношения

Поскольку , то . В результате имеем

,

где

В случае голоморфности с учетом соотношении Коши-Римана приращение функции примет вид .Таким образом,

    приращение голоморфной функции «почти» пропорционально приращению , приращение функции — «почти» линейная комбинация приращении .

Здесь «почти» означает с точностью до . Заметим, что , поэтому . Применяя к обеим частям последнего приближенного равенства сначала операцию модуль, затем аргумент комплексного числа имеем

    геометрически означает локальное растяжение в раз, если . Случай соответствует сжатию. Таким образом, модуль производной означает коэффициент локального растяжения или сжатия. геометрически означает локальный поворот против часовой стрелки на один и тот же угол , то есть разные вектора , начинающиеся в точке , поворачиваются на один и тот же угол. Таким образом, означает на какой угол надо повернуть , чтобы получить направление соответствующее .

Вопрос к экзамену: Гидродинамическая интерпретация голоморфной функции[6].

Рассмотрим установившееся плоскопараллельное течение жидкости. Заметим, что течение жидкости в каждой точке характеризуется ее скоростью. Таким образом, возникает поле скоростей течения жидкости. Течение считается установившимся, если вследствие прошествия достаточно долгого промежутка времени течение «забывает» о причинах возникновения течения. Плоско-параллельное течение характеризуется тем, что в каждом плоском (допустим горизонтальном) сечении поле скоростей одинаково и жидкость из одной плоскости не перетекает на соседнюю. Иначе говоря, течение не зависит от глубины. Поэтому достаточно рассмотреть течение в одном горизонтальном сечении, которую примем за плоскость комплексного переменного . При этом поле скоростей будет плоским и описывается двухкомпонентным векторным полем. Пусть поле скоростей обладает свойством: циркуляция по любому замкнутому контуру равна нулю (смотрите рис.1.а, 1.б, 1.в).

Циркуляция вдоль замкнутой кривой (трубки) векторного поля равна произведению средней касательной скорости на длину кривой (рис.2). В курсе математического анализа констатировалось без объяснения физической сущности, что циркуляция вдоль замкнутого контура равна интегралу векторного поля по кривой, который можно вычислить по формуле Грина также через двойной интеграл по внутренности замкнутого контура:

.

По предположению векторное поле скоростей в нашем случае таково, что циркуляция по любому замкнутому контуру поля равна нулю (нет вихрей). Из приведенной формулы Грина следует выполнение соотношения

Поскольку интегралы по замкнутым кривым равны нулю (поле без вихрей), то интеграл по кривой, соединяющий две точки, не зависит от формы кривой, а является функцией концов кривой.

где — кривые, соединяющие одни и те же точки,

— означает, что ориентация изменена.

Поэтому можно ввести функцию по формуле , так как интеграл зависит только от концевых точек кривой интегрирования. При этом выполняются соотношения , то есть . Поэтому говорят, что поле скоростей в жидкости порождается скалярным полем . Функцию называют потенциалом поля . Итак, поле без вихрей обладает потенциалом.

Сделаем еще одно предположение о течении жидкости. Пусть поток жидкости через каждую замкнутую кривую равен нулю. Это означает, что количество втекающей во внутренность замкнутой кривой равно количеству вытекающей из нее жидкости. То есть нет стоков и источников жидкости. Для произвольной поверхности суммарный поток равен среднему значению нормальной компоненты скорости (отсчитываемой наружу), умноженному на площадь поверхности. В курсе математического анализа без объяснения физической сущности была приведена формула для потока в виде поверхностного интеграла. Причем согласно формуле Гаусса поток можно вычислить и через тройной интеграл.

В нашем случае течение плоское , поэтому предыдущая формула может быть записана в виде . По предположению поток векторного поля через произвольную замкнутую кривую равен нулю ( нет стоков и источников), поэтому выполняется соотношение

Рассуждая также как рассуждали при введении потенциала , можем ввести скалярную функцию . Убеждаемся, что справедливо соотношение

Функцию называют функцией тока траекторий частиц жидкости. Итак, поле скоростей без стоков и источников имеет функцию тока.

Таким образом, установившееся плоскопараллельное течение жидкости без вихрей, без стоков и источников порождает

    потенциал , такой что ; функцию тока , такую что .

Построим теперь комплексную функцию , которая называется комплексным потенциалом поля. Для комплексного потенциала справедливы соотношения Коши-Римана

Следовательно, комплексный потенциал является голоморфной функцией. Обратное также верно. Таким образом, голоморфность функции означает, что эту функцию можно трактовать как комплексный потенциал плоскопараллельного установившегося течения жидкости без вихрей, без стоков и источников.

Лекция 6

Интегрирование функции комплексной переменной

Хорошее определение должно быть посылкой теоремы.

Дж. Глимм

Вопрос к экзамену: Интегрирование функции комплексной переменной.

Интегралы от функции комплексной переменной можно вводить разными способами. Пусть в области определения функции комплексной переменной дана кривая[7] и надо определить интеграл вдоль , который будем обозначать через .

    Первое определение. Формально преобразуем

,

где — реальная и мнимая части , — реальная и мнимая части .

Правая часть последнего равенства может служить определением для левой части, так как в правой части фигурируют известные из курса математического анализа криволинейные интегралы от действительнозначных функций. К примеру,

    Второе определение. Формально преобразуем

Правая часть последнего равенства может служить определением для левой части, так как в правой части фигурируют известные из курса математического анализа криволинейные интегралы от действительнозначных функций.

Упражнение 10. Выразить интеграл вдоль через

Упражнение 11. Выразить интеграл вдоль через

    Пятое определение. Отметим на кривой точки , расположенные на от ее начала к концу. На каждой дуге выберем произвольным образом по точке и вычислим значение . Затем составим интегральную сумму Римана и найдем ее предел при неограниченном размельчении набором точек . Если предел существует и не зависит от способа измельчения и выбора точек , то значение предела берут в качестве определения .

Отметим, что все указанные определения приводят к одному и тому же интегралу. Причем интеграл вдоль обладает естественными свойствами:

    линеен относительно подынтегральной функции; аддитивен относительно объединения кривой интегрирования; интеграл по кривой можно с наперед заданной точностью приближать интегралом по ломаной.

Вопрос к экзамену: Интегральная теорема Коши.

Доказательства приведенных выше свойств интегралов дословно повторяют соответствующие рассуждения из математического анализа, поэтому их здесь не приводим. Лучше остановимся на новых эффектах, которые не наблюдались раньше. Как же эффективно вычислить ? Отметим, что кривая может быть довольно сложной. Коши предложил при вычислении интеграла заменить сложный контур на более-менее простой и только затем вычислять интеграл. При этом правда, может получиться совсем иное значение, чем искомое. А можно ли осуществить замену контуров так, чтобы интеграл сохранял значение? Заметим, что умозрительно можно задаться вопросом: « Нельзя ли сложные подынтегральные функции заменять на простые и затем вычислять интеграл?» Можно, но при этом значения интегралов могут измениться. В то же время на первый вопрос Коши нашел красивый ответ.

Упражнение 12. Доказать, что при фиксированных целых интегралы и равны между собой, то есть единичную окружность при вычислении интеграла можно заменить на квадрат и наоборот.

Упражнение 13. Доказать, что при фиксированном целом справедлива формула

причем единичную окружность можно заменить на окружность произвольного радиуса с центром в начале координат.

Упражнение 14. Доказать, что при фиксированном целом справедлива формула

То есть ответ при зависит от принадлежности или непринадлежности точки кругу . Используя упражнения 12, 13, приведем доказательство упражнения 14.

Доказательство. Не умаляя общности можно считать, что . Допустим, что лежит вне единичного круга. При разложим

    в ряд Тейлора при по биному Ньютона при

Отрицательных степеней в разложениях нет, поэтому почленное интегрирование с учетом упражнения 13 позволяет утверждать, что соответствующий интеграл равен нулю. Теперь рассмотрим случай . При разложим в ряд Тейлора

Когда ,то отрицательные степени начинаются от -2,-3 и так далее. Согласно упражнению 13 интеграл от правой части последнего равенства будет равен нулю. Если , то согласно упражнению 13 получим ровно . Положительные рассматриваются аналогично. Что и требовалось доказать.

Следствие из упражнения 14. При произвольно целом в интеграле можно заменять единичную окружность на любую другую окружность, содержащую точку нуль. При этом значение интеграла сохраняется. Те кривые, которые при замене сохраняют значения интегралов, называют гомотопными между собой кривыми. В данном следствии окружности, содержащие в качестве внутренней точки начало координат, будут гомотопны между собой. Приведем строгое определение гомотопных кривых.

Случай кривых с общими концами. Допустим, что в области даны две кривые , соединяющие две точки . Говорят, что гомотопно относительно области , если непрерывно деформируя в можно получить ( смотрите рис.3)

.

Если уравнение кривой имеет вид , а уравнение кривой имеет вид , то уравнения промежуточных деформированных кривых будем писать в виде при . Иногда вместо записи удобно писать . В таком случае непрерывная деформация в области означает, что функция непрерывна как функция двух переменных и . На практике в возможности непрерывной деформации относительно некоторой области можно убедиться без всяких формул типа .

Случай замкнутых кривых. Две замкнутые кривые и из области называются гомотопными друг другу относительно области , если непрерывная в деформация одной из них совмещается с другой кривой. Если замкнутую кривую, оставаясь в , можно непрерывно деформировать в точку, то будем говорить кривая в стягивается в точку или кривая в гомотопна нулю.

Гомотопия в кривых и обозначается . Теперь мы готовы сформулировать

Теорема 6. (Коши) Интеграл от голоморфной в области вдоль любых гомотопных в кривых принимает одно и то же значение. Краткая формулировка:

Здесь и всюду в дальнейшем будет обозначать класс голоморфных в функций. Приведем различные эквивалентные формулировки теоремы Коши. Как всегда означает границу области . Когда граница — связное множество, то будем говорить, что — односвязная область.

Теорема 7. (Коши для односвязной области) Интеграл от голоморфной в односвязной области функции вдоль любого замкнутого контура равен нулю.

Кстати, замкнутые кривые часто представляют границы областей. Поэтому справедлива

Теорема 8. (Коши для многосвязных областей)

Утверждение 4. Из теоремы 7 вытекают теоремы6, 8.

Пусть . Доказывать надо только для незамкнутых контуров. Рассмотрим замкнутый контур , где — совпадает с контуром , только его ориентация изменена. Тогда найдется односвязная подобласть области , которая содержит замкнутый контур . Из теоремы 7, примененной для односвязной подобласти , следует, что . Отсюда получаем

. Что и доказывает теорему 6.

Пусть . Если граница — связное множество, то -раздутие

подобласти представляет односвязную область и — замкнутый контур в этом раздутии. Из теоремы 7 для этого раздутия следует, что .

Если граница — несвязна, то достраиваем мосты, соединяющие несвязные компоненты границы. Причем по одному и тому же мосту производится обход дважды и в разных направлениях. С помощью построенных мостов можно получить замкнутый контур, лежащий в односвязной области. Затем применим теорему 7. Откуда получаем теорему 8 из теоремы7. Студенту достаточно запомнить этот прием для

кольца. На рисунке (смотри на следующей странице) кольцо подобласть области и его граница состоит из двух окружностей. Запомните: граница считается ориентированной, то есть она ориентирована так, что при ее обходе область остается слева. Эти окружности соединяем мостом и обходим его дважды в разных направлениях. Тогда можно получить ориентированную замкнутую линию гомотопную в области контуру (смотри рисунок). Контур можно считать принадлежащим некоторой односвязной области и применить теорему 7 к этой области. Тогда из теоремы 7 будет следовать теорема 8, так как интегралы по дважды обходимому мосту взаимно уничтожаются.

Пояснения к данному рисунку даны выше.

На рисунке не отображена ориентация границы .

Покажите: как надо деформировать в области границу с двойным обходом моста, чтобы получить контур .

Итак, согласно утверждению 4 достаточно доказать теорему 7.

Схема доказательства теоремы 7 (для тех кто довольствуется тройкой).

1 шаг. Интеграл по непрерывному контуру можно приближенно заменить интегралом по специально подобранной ломанной.

2 шаг. Интеграл по ломанной в точности совпадает с суммой интегралов по некоторому набору треугольников.

3 шаг. Интеграл по достаточно большому треугольнику в точности совпадает с суммой интегралов по некоторому набору треугольников очень малых размеров.

4 шаг. Интеграл вдоль треугольника достаточно малого периметра от произвольной голоморфной функции можно приблизить интегралом по тому же треугольнику от некоторой линейной функции.

5 шаг. Интеграл от линейной функции вдоль любого треугольника равен нулю. Надо аккуратно вычислить этот интеграл.

6 шаг. Первый вывод из предыдущих шагов: исходный интеграл почти нуль. Более детальный анализ позволяет заключить, что он точно равен нулю. Теорема 6 доказана (на тройку).

Грамотное математическое оформление 1-6 шагов. Пусть замкнутый контур лежит в односвязной области . В области возьмем компакт (замкнутое и ограниченное множество) , содержащий контур . Голоморфная на функция будет непрерывной на компакте , тогда по теореме Кантора она на равномерно непрерывна. . На контуре выберем упорядоченный набор точек так, чтобы расстояние (по дуге) между соседними точками было меньше . Обозначим через ломанную, проходящую через точки . Оценим разность

Таким образом, 1 шаг реализован. Второй шаг на рисунке (иногда лучше рисовать, а не разговаривать). Внутри произвольно выбрана жирная точка, которая соединена со всеми вершинами. В результате разбит на треугольники. Ориентация сторон треугольников согласована с ориентацией . Тогда внутренние стороны треугольников обходятся дважды в разных направлениях. Их интегральный вклад нулевой.

Таким образом, 2 шаг реализован. Молча реализуем третий шаг (смотри рисунок).

Таким образом, периметр треугольника уменьшился в четыре раза. Никто не может возразить, если я дальше буду продолжать дробление треугольников.

Реализация четвертого шага. Поскольку подынтегральная функция — голомофная в , то в окрестности любой точки из функция хорошо приближается линейной функцией:

Пусть дан треугольник, который можно вписать в окружность радиуса . Обозначим центр описанной вокруг данного треугольника окружности через . Тогда этот треугольник лежит внутри круга , причем можно считать . Поэтому верна оценка

Аккуратные вычисления пятого шага ( их надо привести на экзамене) дают

В силу произвольности и из того, что левая часть последнего неравенства не зависит от , получим Отсюда сразу следует . Тогда из неравенств первого шага имеем . В силу произвольности и из того, что левая часть последнего неравенства не зависит от , получим .

Лекция 7

Интегральная формула Коши

Математик обычно получает теорему с помощью интуиции; он обнаруживает правдоподобное заключение и начинает работать над созданием доказательства.

Готфри Гарольд Харди

Вопрос к экзамену: Интегральная формула Коши.

К сожалению, индийский математик Рамануджан[8] не знал теоремы Коши и всего того, что представляет содержание дальнейших лекций. На самом деле, теорема Коши дает мощный аппарат, используемый в различных разделах науки[9]. Теорема Коши говорит о равенстве интегралов . Поэтому по своему желанию можно вычислять либо левую, либо правую часть равенства. Обычно выбирают ту, которая легче находится. Применим подобные рассуждения для вычисления интеграла , когда — голоморфная на всей комплексной плоскости функция, а — произвольный замкнутый контур

·  Если точка то подынтегральная функция будет голоморфной в некотором — раздутии . Тогда по теореме 7 .

·  Если точка то подынтегральная функция будет голоморфной в некоторой проколотой окрестности точки . Каждая окружность с центров в точке гомотопна контуру , так как, не затрагивая точку , можно контур деформировать в указанную окружность. По теореме 6 имеем . Поскольку левая часть последнего равенства не зависит от , то можно записать предельное соотношение

Таким образом, надо найти правую часть последнего равенства.

Следовательно, при имеем формулу , которую называют интегральной формулой Коши. Продифференцировав по обе части приведенной формулы, получим представление для производной . Можно продолжить дифференцирование.

Обсуждение интегральной формулы Коши.

·  Из следует, что внутренние значения голоморфной функции можно находить (восстановить) по значениям на границе . Подобные проблемы восстановления функции по граничным значениям возникают в теории краевых задач.

·  Вспомним, что рациональная функция представляется в виде суммы простейших дробей. К примеру, когда , то справедливо разложение . Напомню, что интеграл – это предел сумм. Тогда интегральная формула Коши представляет полный аналог разложения рациональной функции (роль играет ). Следовательно, голоморфная функция обладает многими свойствами рациональных функций.

·  Поскольку интегральную формулу Коши можно по параметру неограниченное число раз дифференцировать, то голоморфная функция бесконечно дифференцируема. Это неожиданное следствие!!! В курсе математического анализа встречались функций дифференцируемые только конечное число раз. Здесь таких нет!!!

·  Интегральная формула Коши вместо вычисления интеграла разрешает вычислять значения голоморфной функции или значения ее производной. Действительно

Вопрос к экзамену: Локальное разложение голоморфных функций в степенные ряды.

Из интегральной формулы Коши вытекает, что значения голоморфной функции можно «собрать» из простейших дробей вида , где . С другой стороны, дробь вида при разлагается в степенной ряд. Поэтому интуитивно понятно, что голоморфная функция также (где-то) разлагается в степенной ряд. Аккуратно реализуем интуицию в виде строгих математических выкладок.

Теорема 9.

Упражнение 15. Сформулируйте словесную формулировку теоремы 9.

Замечание для информативности. В теореме 9 разложение осуществляется в некотором круге . То есть разложение локально, так как нет оценки радиуса .

Вопрос для творческих натур. Анализируя доказательство теоремы 9, попробуйте найти максимально возможный радиус .

Доказательство теоремы 9. 1 шаг. Разложение простейшей дроби вида .

2 шаг. Разложение голомофной функции в ряд Тейлора в окрестности фиксированной точки. Пусть — фиксированная точка из области . Выберем круг с центром в точке так, чтобы .

По интегральной формуле Коши найдем значение функции в произвольной точке круга . Теперь используем результат первого шага. , так как выполняется неравенство .

Обе части последнего равенства проинтегрируем согласно интегральной формуле Коши

Отсюда следует, что , где коэффициенты

Теорема 9 доказана.

Уточним теорему 9 в следующем направлении.

Теорема 10. Указанное в теореме 9 разложение единственно.

Доказательство. Допустим, что в круге функция разлагается в ряды и . Надо показать, что . К примеру, . Точно также имеем . Дальше по индукции…Теорема 10 доказана.

Следствие 1 из теорем 9, 10. Голоморфная функция бесконечное число раз дифференцируема в области голоморфности.

Это следует из того, что таким свойством обладает сумма степенного ряда Тейлора.

Следствие 2 из теорем 9, 10. Производные голоморфной функции имеют представления и так далее. Для обоснования достаточно детально проанализировать доказательства теорем 9 и 10.

Следствие 3 из теорем 9, 10. Пусть — нуль кратности голоморфной функции , тогда в некоторой окрестности точки справедливо представление Безу , где — голоморная функция, причем

Доказательство. Так как , то из разложения функции в ряд Тейлора в окрестности точки и следствия 2 имеем . Остается обозначить через . Следствие доказано.

Следствие 4 из теорем 9, 10. Нули голоморфной функции изолированы.

Это следует из следствия Безу. Так как множитель не может обращаться в нуль в некоторой окрестности точки . Тогда произведение в указанной окрестности может обращаться в нуль только за счет первого множителя , который имеет изолированный нуль .

Следствие 5 из теорем 9, 10. Нули голоморфной функции не могут иметь предельных точек, принадлежащих области голоморфности.

Доказательство. Предположим, что существует сходящаяся последовательность нулей , причем их предел где — область голоморфности функции . В силу непрерывности функции имеем , что противоречит изолированности нуля .

Лекция 8

Теоремы Лиувилля и Мореры

Прежде чем писать, посмотри, как прекрасен девственно чистый лист бумаги.

Станислав Ежи Лец

Вопрос к экзамену: Целые функций. Теорема Лиувилля.

До сих пор мы изучали функции, которые являлись голоморфными в некоторой открытой части комплексной плоскости. Теперь исследуем свойства голомофных на всей комплексной плоскости. Голоморфная на всей комплексной плоскости функция называется целой функцией. Приведем некоторые свойства целой функции:

·  Разлагается в ряд Тейлора с бесконечным радиусом сходимости.

·  Нули изолированы без конечных предельных точек.

Указанные свойства – другая формулировка теорем 9 , 10 и их следствий. Оказывается, что непостоянных ограниченных целых функций не существует. Это удивительный факт, если вспомнить, что

    тригонометрические функций синус и косинус разлагаются в ряды Тейлора с бесконечным радиусом, тригонометрические функций синус и косинус ограничены на всей числовой прямой.

Теорема 11. (Лиувилль Жозеф[10]) Ограниченная целая функция – постоянная функция.

Доказательство. Согласно теореме 9 в круге произвольного радиуса целую функцию можно разложить в ряд Тейлора . Оценим модуль коэффициента , если . Так как радиус произвольное число и коэффициент не зависит от , то переходя к пределу в неравенстве , получим . Точно также оценивается . Продолжая подобные рассуждения получим, что . В результате для всех . Теорема доказана.

Следствие из теоремы 11. Целая функция является неограниченной функцией.

Доказательство. Так как , то следствие сразу же вытекает из теоремы 11.

Упражнение 16. На каких лучах комплексной плоскости целая функция неограниченно растет?

Теорема 12. (Лиувилль Жозеф) Целая функция, модуль которой растет не быстрее некоторой степени модуля своего аргумента, представляет многочлен.

Доказательство. Согласно теореме 9 в круге произвольного радиуса целую функцию можно разложить в ряд Тейлора . Оценим модуль коэффициента , если . Так как радиус произвольное число и коэффициент не зависит от , то переходя к пределу в неравенстве , получим . Точно также оценивается . Продолжая подобные рассуждения получим, что . В результате для всех . Теорема доказана.

Следствие из теоремы 12. Целая функция, модуль которой растет не быстрее некоторой

степени модуля своего аргумента, может иметь только конечное число нулей.

Обсуждение следствия. Многочлен степени имеет не более нулей, то есть чем быстрее на бесконечности растет многочлен, тем больше у него нулей. К примеру, линейная функция имеет один нуль, а квадратный трехчлен – два нуля. Квадратный трехчлен на бесконечности растет как вторая степень, а линейная функция на бесконечности растет как первая степень независимой переменной. Оказывается, подобное утверждение верно и для целых функций, если исключить целые функций вида . Иначе говоря, только экспонента не имеет нулей!!! Рост многочленов определяется их степенью. Как же определить порядок роста целой функции? Если существует положительное число такое, что при всех комплексных выполняется неравенство , то целая функция называется конечного порядка роста. Целые функций конечного порядка роста широко применяются в различных разделах математики и физики[11] . Среди казахстанских математиков в своих исследованиях целые функций эффективно использовал Т. И.Аманов.

Вопрос к экзамену: Теорема Мореры[12]. Интеграл типа Коши. Некоторые итоги.

Вспомним пройденный материал. Для наглядности приведем диаграмму.

Центральным понятием является понятие голоморфной в области функции. Стрелки на диаграмме показывают следствия из голоморфности функции. В некотором смысле верны также обратные утверждения. В предыдущих лекциях все обратные утверждения доказаны, кроме ИТК и Th M ( на диаграмме жирные линии).

Интеграл типа Коши (ИТК). Возьмем контур и проинтегрируем непрерывную функцию вдоль этого контура . Такой интеграл назовем интегралом типа Коши. Оказывается, что указанный интеграл типа Коши при представляет голоморфную функцию. Таким образом, мы получили еще один способ задания голоморфных функций. Более полную информацию о ИТК можно получить на спецкурсах кафедры математического анализа.

Теорема 13 (Морера ( Th M)). Если функция непрерывна в области и интеграл от нее по границе любого треугольника равен нулю, то .

Доказательство. Для любой точки из области построим круг . Введем функцию по формуле . Покажем, что . Поэтому рассмотрим соответствующее приращение функции Тогда отношение приращений примет вид

Таким образом, построенная функция имеет производную, то есть является голоморфной функцией. Ее производная — также голоморфная функция. Теорема доказана.

Замечание о терминологии. Аналитическая функция – функция представимая (локально) в виде ряда Тейлора. Голоморфная функция — дифференцируемая функция на открытом множестве. На первой стадии знакомства с ТФКП ничего не случится, если эти понятия отождествлять. В дальнейшем будем изучать полные аналитические функций, где надо быть поосторожней.

Лекция 9

Изолированные особые точки

Если бы К. Вейерштрассе публиковал своевременно свои результаты, то вместо рядов Лорана мы сегодня изучали ряды Вейерштрассе.

Лектор.

Ряды Лорана. Классификация изолированных особых точек однозначного характера.

Вопрос к экзамену: Ряды Лорана.

Степенные ряды с целыми показателями степеней называются рядами Лорана. Ряды Лорана – это обобщение рядов Тейлора. Изучим свойства рядов Лорана. Итак, рассмотрим ряд вида . Впервые мы встречаемся с ситуацией, когда ряд не имеет ни начала, ни конца. Как же определить сходимость такого ряда? Можно поступать по разному. К примеру, в качестве суммы ряда можно взять один из пределов

Мы выбираем в качестве суммы ряда Лорана последнее определение

Теорема 14. (П. Лоран, 1843) Сумма ряда Лорана голоморфна в некотором кольце.

Доказательство. Так как

то надо изучить отдельно функций . Из того, что — сумма ряда Тейлора, то — голоморфная функция в некотором круге с центром в точке . Обозначим , тогда функция — голоморфная функция в круге на — плоскости. На — плоскости это означает, что функция — голоморфная функция вне круга . Следовательно, на пересечении кругов сумма будет голоморфной функцией.

Теперь докажем обратное утверждение.

Теорема 15. Голоморфная в открытом кольце функция разлагается в этом кольце в ряд Лорана.

Доказательство. Пусть функция голоморфна в кольце . По произвольной точке из кольца выберем числа так, чтобы . Тогда согласно ИФК имеем

Разложение в степенной ряд для дословно повторяет доказательство теоремы 9, так как в этом случае . Таким образом, Аккуратно запишем разложение в степенной ряд для .

Отсюда имеем соотношение .Теорема доказана.

Следствие из теоремы 14. ,то есть значение интеграла совпадает с одним из коэффициентов разложения в ряд Лорана. Следовательно, вместо вычисления интеграла можно разлагать в ряд Лорана. Это очень важное следствие.

Вопрос к экзамену: Классификация изолированных особых точек однозначного характера.

Точки, в которых отображение голоморфно или аналитично, назовем регулярными точками отображения. Там, где нарушается голоморфность или аналитичность, назовем особыми точками отображения. Особая точка будет изолированной особой точкой данного отображения, если существует проколотая окрестность особой точки, состоящая из регулярных точек. Изолированные особые точки делим на вида: однозначного и многозначного характеров. Здесь мы изучаем только однозначные функций[13]. Итак, пусть в области задана однозначная функция . Допустим, что голоморфна во всех точках области , кроме точек . Считаем, что среди особых точек нет одинаковых. Тогда каждая из указанных точек будет изолированной особой точкой функции . Возьмем некоторую проколотую окрестность точки и согласно теореме 15 разложим в

ряд Лорана

Сумму членов с отрицательными степенями назовем главной частью разложения Лорана, а оставшиеся слагаемые дадут правильную часть. Изолированная особая точка в зависимости от количества слагаемых в главной части считается:

устранимой изолированной особой точкой, если главная часть отсутствует, полюсом порядка , если главная часть содержит только конечное число слагаемых и показатель наименьшей отрицательной степени в главной части, существенно особой точкой, если в главной части бесконечно много слагаемых.

Примеры.

1. 

2. 

3. 

4. 

В последнем примере показана неизолированная особая точка. Такие точки исследуются в продвинутых курсах ТФКП.

Определение характера изолированной особой точки, исходя из соответствующего разложения Лорана, не всегда эффективно. Ниже приведены утверждения, применение которых менее трудоемко.

Теорема 16. (Критерии устранимой особой точки) Пусть — изолированная особая точка функции . Следующие предложения эквивалентны:

— устранимая особая точка, существует конечный предел

Доказательство. Тогда главная часть разложения Лорана отсутствует, предел правильной части всегда существует и конечен.

Тогда — ограниченная функция в окрестности , что противоречит существованию в разложении Лорана отрицательных степеней .

Теорема 17. (Критерии полюса) Пусть — изолированная особая точка функции . Следующие предложения эквивалентны:

— полюс, существует бесконечный предел

Доказательство. Тогда главная часть разложения Лорана содержит конечное число слагаемых вида . Предел таких слагаемых равен бесконечности, а предел правильной части всегда существует и конечен. Тогда их сумма равна бесконечности.

Введем функцию . Тогда для либо регулярная точка, либо изолированная особая точка. Причем существует конечный предел . Из теоремы 16 вытекает, что — устранимая особая точка функции . Доопределим функцию так: . Тогда[14] голоморфная функция в окрестности , причем — нуль функции . Из следствия 3 теоремы 9 вытекает представление . Отсюда имеем представление , причем второй множитель в правой части последнего равенства представляет локально голоморфную функцию и поэтому в его разложении нет отрицательных степеней . В соответствующем разложении Лорана функции отрицательные степени могут возникать только за счет множителя . Так как число — конечное число, то таких степеней также конечно.

Теорема 18. (Критерии существенно особой точки) Пусть — изолированная особая точка функции . Следующие предложения эквивалентны:

— существенно особая точка, не существует предела

Доказательства не требует или методом от противного.

Упражнение 17. Сформулируйте критерии полюса порядка 17.

Упражнение 18. Решите предыдущие примеры, используя приведенные критерий.

Лекция 10

Вычеты

Весь предшествующий опыт убеждает нас в том, что природа представляет собой реализацию простейших математически мыслимых элементов.

Эйнштейн

Вычеты однозначной аналитической функции. Поведение функции в окрестности существенно особой точки. Аналитическое продолжение по Коши.

В данной лекции показано как на практике применяются результаты предыдущих занятий. Может быть, эта лекция — одна из самых важных лекции — поможет сдать экзамен и зачет по данному предмету.

Вопрос к экзамену: Вычеты однозначной аналитической функции.

Вычеты в математике встречаются в двух совершенно разных разделах математики. В теории чисел – это вычет целого числа по модулю другого целого числа. К примеру, число 24 есть вычет числа 3 по модулю 7. Мы будем изучать вычеты, введенные Коши (1814). Вычет функции в изолированной особой точке по определению равен значению интеграла взятого в положительном направлении по окружности достаточно малого радиуса с центром в изолированной особой точке этой функции. Обозначение: . В случае устранимой особой точки и полюса вычеты вычисляются по формулам:

, если — конечная устранимая особая точка, , если — простой полюс, , если — полюс порядка два, , если — полюс порядка три, , если — полюс порядка четыре и так далее.

Доказательство формулы 4 (остальные доказываются аналогично). Так как — полюс порядка три, то в окрестности справедливо разложение Лорана . Нам надо вычислить интеграл . Вместо подставим разложение Лорана и учтем результаты упражнения 14, тогда имеем . Таким образом, вычет в точности совпадает с одним из коэффициентов разложения Лорана. Найдем его.

Остается найти требуемый предел. Формула доказана.

Пример

Теперь займемся вычислением более сложного интеграла

Теорема 19. Вышеприведенный интеграл равен сумме внутренних вычетов подынтегральной функции, то есть

Доказательство. Вокруг каждой особой точки функции , лежащей внутри замкнутого контура, опишем окружность достаточно малого радиуса. Потребуем, чтобы они не пересекались между собой и не пересекали контур . Тогда по теореме 8 имеем

Здесь мы учли ориентацию маленьких окружностей. Теорема доказана.

До сих пор в качестве особых точек выступали конечные точки. А как быть: если особой будет бесконечно удаленная точка? Пусть функция имеет особенность в бесконечности, то есть . Чтобы исследовать характер особой точки достаточно сделать замену . Тогда функция будет иметь изолированную особенность при . Если — устранимая особая точка функции , то — также устранимая особая точка функции . Характер особенности сохраняется. Полюс переходит в полюс и так далее. Вычет в бесконечности по определению есть интеграл , где контур интегрирования ориентирован по ходу часовой стрелки. Не надо думать, что вычет в бесконечно удаленной устранимой особой точке всегда равен нулю. То есть вышеприведенная формула 1 для вычетов верна в случае конечных точек.

Теорема 20. Если функция имеет только конечное число особых точек однозначного характера, то сумма всех вычетов этой функции равна нулю.

Доказательство очевидно. На самом деле, мы специально подобрали определение вычета в бесконечности, чтобы выполнялась эта теорема.

Упражнение 19 к теореме 20. А что было бы, если взять в качестве вычета на бесконечности один из коэффициентов главной части разложения Лорана?

Вопрос к экзамену: Поведение функции в окрестности существенно особой точки.

Если в окрестности полюса и устранимой особой точки поведение функции достаточно регулярно (существует предел), то совсем иначе устроена функция в окрестности существенно особой точки

Теорема 21. (Ю. Сохоцкий – Ф. Казорати) Если является существенно особой точкой функции , то для любого конечного или бесконечного найдется последовательность точек сходящаяся к особой точке и такая, что

Теорема 22. ( Э. Пикар, 1879) В любой окрестности существенно особой точки любая функция принимает все значения, за исключением, быть может, одного.

Теорема Пикара существенно сильнее теоремы 21. Ее часто называют большой теоремой Пикара, так как имеется еще и малая теорема. Пока мы не готовы доказать теорему 22, так как для этого надо знать модулярную функцию. Пока докажем только теорему 21.

Доказательство теоремы 21. Так как в окрестности существенно особой точки функция неограниченна, то теорема верна при . Пусть теперь — конечное число. Предположим противное: найдется проколотая окрестность особой точки. функция не принимает значения . Введем функцию по формуле ограничена в окрестности особой точки. Чего не может быть, так как при таком построении сущесвенно особая точка не может стать устранимой особой точкой. Теорема доказана.

Упражнение 20. Проверьте теорему 21 для функции при . Надо предъявить соответствующие последовательности.

Упражнение 21. Проверьте теорему 22 для функции при . Надо предъявить бесконечно много точек, где значение экспоненты принимает требуемое значение. Найти исключительное значение.

Вопрос к экзамену: Аналитическое продолжение по Коши.

Факториал определен на натуральных числах, то есть . Как определить значения факториала от нецелых чисел? Это делает гамма-функция Эйлера. Область определения гамма-функции шире области определения факториала, сужение на гамма-функции совпадает с факториалом. Поэтому говорят, что гамма-функция есть продолжение факториала. Можно ли дальше расширять область определения, сохраняя ранее определенные значения функции? Осуществим подобное продолжение факториала, следуя методу Коши.

1 шаг. Продолжение в полуплоскость. Гамма-функция Эйлера при задается несобственным интегралом . Покажем, что та же формула позволяет продолжить гамма-функцию в область . Для этого надо доказать, что интеграл сходится при . Это следует из оценок:

Итак, факториал корректно определен и для комплексных чисел.

2 шаг. Продолжение в вертикальную полосу. Следующие преобразования позволяют расширить область определения гамма-функции. При верны равенства

Итак, гамма-функция при представляет сумму трех функций

    Область определения функции есть Область определения функции есть , так как функция ограничена на отрезке Область определения функции есть

Таким образом, сумма определена на пересечении . Итак нам удалось расширить область определения гамма-функции.

3 шаг. Продолжение в вертикальную полосу. Следующие преобразования позволяют расширить область определения гамма-функции. При верны равенства

Последняя сумма определена на области . Итак, гамма-функция продолжается на области .

Можно продолжать намеченный процесс неограниченно долго. В результате имеем

Утверждение 5. Гамма- функция Эйлера может быть продолжена на область , причем это продолжение представляет голоморфную функцию.

Доказательство. Надо доказать, что каждая из функции — голоморфна в своей области определения. Это следует из того, что

    голоморфна в своей области определения, голоморфна в своей области определения

Утверждение доказано.

Замечание. Метод улучшения сходимости, который здесь применялся, называется методом Коши.

Замечание. На самом деле, продолжение гамма-функции не только сохраняется ранее определенные значения функции, но и не ухудшает ее аналитические свойства. То есть мы получили аналитическое продолжение. Таким образом, задача аналитического продолжения функции заключается в продолжении с сохранением ее аналитичности (голоморфности).

Пример неаналитического продолжения.

Функция является продолжением функции . Так как разлагается в ряд Тейлора, то — аналитическая функция. В то же время функция — не может быть разложена (даже локально) в ряд Тейлора, так как она не является непрерывной функцией на комплексной плоскости. То есть сохраняет значения функции , но не сохраняет ее аналитические свойства.

Лекция 11

Аналитическое продолжение элементов

Вейерштрассе Карл Теодор Вильгельм (31.10.1814 – 19. 02.1897) – немецкий математик. Центральное место в работах Вейерштрассе занимает теория аналитических функций, в основу которой он кладет степенные ряды.

Математический энциклопедический словарь

Понятие аналитического продолжения. Аналитическое продолжение по Вейерштрассе. Теорема о монодромии. Изолированные особые точки многозначного характера.

Вопрос к экзамену: Понятие аналитического продолжения.

До сих пор нас интересовали локальные свойства функции. Когда изучают глобальные свойства (к примеру: надо найти максимальную область определения локально заданной функции), однозначность в определении функции становится обременительным требованием. То есть возникает потребность изучать многозначные «функций». Как работать (оперировать) с многозначными функциями?

Пример. Какой смысл надо вложить в сумму , если каждое слагаемое принимает два значения? Если складывать как совокупности, то эта сумма будет уже трехзначной. Складывать лишь значения с одинаковым знаком? Но у комплексных чисел нет понятия положительного или отрицательного: пусть ; какие знаки слагаемых в сумме одинаковы[28]?

В данном пункте излагается подход Вейерштрассе. Первоначально на некоторой области задается голоморфная функция .

Задача 1. Найти аналитическое продолжение , определенной уже на , подчиненное условиям:

сужение на совпадает с первоначальной функцией , — голоморфная функция на .

Задача 2. Найти аналитическое продолжение , определенной уже на области , подчиненное условиям:

сужение на совпадает с сужением первоначальной функцией на , — голоморфная функция на .

Аналитическое продолжение в виде пары , определяемое в задачах 1 или 2, называется непосредственным аналитическим продолжением элемента .

Задача 3. Найти аналитическое продолжение , определенной уже на области , подчиненное условиям:

существует последовательность элементов (пар) элемент является непосредственным аналитическим продолжением элемента элемент совпадает с элементом .

Элемент , определяемый как решение задачи 3, называют аналитическим продолжением элемента .

Пример 1. Продолжение гамма-функции, полученное выше, есть решение задачи 1. При этом начальным элементом надо считать пару:

Пример 2. Пусть в области определена функция по формуле . Тогда пара является аналитическим продолжением элемента , если

· 

· 

Действительно, выберем элемент из соотношений:

Ясно, что элемент является непосредственным аналитическим продолжением элемента . В то же время элемент является непосредственным аналитическим продолжением элемента . То есть существует промежуточный элемент , который замыкает последовательность элементов , удовлетворяющих условию 2 задачи 3.

Пример 3. Пусть в области определена функция по формуле . Тогда пара является аналитическим продолжением элемента , если

· 

· 

Выберем и вычислим значение функции . Если , то значение функции . С другой стороны, понятно, что , хотя . То есть функция не является аналитическим продолжением функции , так как теряется однозначность (перестает быть функцией). В то же время элемент является аналитическим продолжением элемента потому, что , и . То есть для элементов однозначность сохраняется.

Вопрос к экзамену: Аналитическое продолжение по Вейерштрассе.

Идея продолжать элементы, а не функции принадлежит К. Вейерштрассе. Как же строить последующие элементы, зная предыдущие? Ответ Вейерштрассе: надо разлагать в степенные ряды функцию из заданного элемента, выбирая тем или иным образом их центры. То есть следующий элемент строится с помощью разложения в ряд Тейлора предыдущего элемента. Уточним сказанное на примерах.

Пример 1. Пусть в области определена функция по формуле . Чтобы построить непосредственное аналитическое продолжение элемента выберем произвольно точку и разложим в ряд Тейлора функцию по степеням . В результате имеем , причем радиус сходимости . Это следует из утверждения 2. При выборе точки надо стараться, чтобы получился круг с возможно большим радиусом . То есть надо выбирать как можно дальше от единицы. Итак, следующий элемент имеет вид :

Указанный процесс продолжаем. В результате получим аналитическую функцию . Шире не продолжить!!!

Пример 2. Пусть в области определена функция по формуле . Чтобы построить непосредственное аналитическое продолжение элемента выберем произвольно точку и разложим в ряд Тейлора функцию по степеням . В результате имеем , причем радиус сходимости . Это следует из утверждения 2. Как устроены особые точки функции ? Во-первых, является особой точкой функции. Запишем тождество , из которого следует, что — особые точки . Из тождества следует, что являются особыми точками. В общем, особые точки образуют плотное множество на единичной окружности. Поэтому . Итак, следующий элемент имеет вид :

Заметим, что . То есть не удается расширить область определения первоначального элемента. Шире не продолжить!!!

Множество всех возможных аналитических продолжений фиксированного элемента образует полную аналитическую функцию. В примерах 2 и 3 приведены полные аналитические функции.

Вопрос к экзамену: Теорема о монодромии.

Итак, мы научились продолжать элементы. Но это элементы, а не функций. Иногда от элементов можно переходить к функциям. Так можно поступать, когда нет неоднозначности. В примерах 1, 2, 3 возникают однозначные аналитические функций, поэтому нет необходимости рассматривать продолжение элементов. Как узнать, когда возникают однозначные продолжения? На этот вопрос отвечает следующая терема.

Теорема 23. (теорема о монодромии) Если некоторый элемент аналитически продолжаем вдоль любого пути , принадлежащего односвязной области , то определяемая продолжениями элемента в области аналитическая функция однозначна в этой области.

Доказательство. Возьмем некоторую точку из и соединим кривой в с произвольной точкой из . В силу односвязности любые две кривые , ведущие в из точки в , гомотопны, а по условию элемент продолжаем при произвольном выборе вдоль гопотопных кривых. центров разложения. Покажем, что получаемые при этом элементы с центром в точке совпадают.

Можно считать, что кривые достаточно близко расположены друг к другу. Иначе проведем промежуточные деформированные кривые. Рассмотрим пару элементов , где центр разложения в круге лежит на , а центр разложения в круге лежит на . Из-за близости круги и пересекаются, а тогда степенные ряды на общей части совпадают с предыдущим разложением. Следовательно, они дают одну и ту же голоморфную функцию. Теорема доказана, так как эти рассуждения справедливы для любых элементов.

Пример 1. Однозначные ветви корня -той степени. Под однозначной ветвью многозначной функции понимаем однозначную функцию, о которой говорится в теореме о монодромии. В качестве односвязной области возьмем плоскость с разрезом, начинающимся в нуле и простирающимся непрерывным образом до бесконечности. Вспомним, что значения корня -той степени определяются по формуле . Многозначность возникает за счет того, что может получить приращение кратное . В выбранной нами области таких приращений у нет. Поэтому применима теорема о монодромии, то есть достаточно зафиксировать целочисленный параметр .

Пример 2. Однозначные ветви аналитической функции . Под однозначной ветвью многозначной функции понимаем однозначную функцию, о которой говорится в теореме о монодромии. В качестве односвязной области возьмем плоскость с разрезом, начинающимся в нуле и простирающимся непрерывным образом до бесконечности. Вспомним, что значения корня -той степени определяются по формуле . Многозначность возникает за счет того, что может получить приращение кратное . В выбранной нами области таких приращений у нет. Поэтому применима теорема о монодромии, то есть достаточно зафиксировать целочисленный параметр .

Вопрос к экзамену: Изолированные особые точки многозначного характера.

Точка называется изолированной точкой аналитической функции , если существует проколотая окрестность точки такая, что некоторый элемент , принадлежащий этой функции, продолжается вдоль любого пути из .

Пусть — изолированная особая точка аналитической функции и — вышеуказанная проколотая окрестность точки . Возьмем произвольный замкнутый контур из и содержащий точку внутри. Возможны два случая:

·  Если обход приводит к элементу, отличному от исходного, то точка называется особой точкой многозначного характера или точкой ветвления.

·  Если обход не меняет исходного элемента функции, то точка называется особой точкой однозначного характера.

В первом случае возможен вариант — -кратный обход приводит к исходному элементу. В этом случае точка называется точкой ветвления конечного порядка. Порядок ветвления – это минимальное из возможных .

Упражнение 22. — имеет в точках точки ветвления порядка . Аккуратно обосновать.

Упражнение 23. имеет в точке устранимую особенность, а в точке — существенно особую точку. Обосновать.

Упражнение 24. Определить характер особых точек

В окрестности точки ветвления конечного порядка справедливы разложения Пюизе.

Теорема 24. (Пюизё, 1850) В некоторой проколотой окрестности точки ветвления конечного порядка аналитическую функцию можно разложить в ряд по нецелым степеням

Доказательство. Замена позволяет точку ветвления порядка свести к особой точке однозначного характера. Дальше надо использовать разложение Лорана.

Лекция 12

Римановые поверхности

Риман Георг Фридрих Бернхард (17.09.1826 – 20. 07. 1866) – немецкий математик. В 1851 году защитил докторскую диссертацию «Основы общей теории функций одной комплексной переменной», где положил начало геометрическому направлению теории аналитических функций.

Математический энциклопедический словарь

1.  Понятие римановой поверхности.

2.  Принцип аргумента.

Вопрос к экзамену: Понятие римановой поверхности.

В начале приведем пример аналитического продолжения функции, приводящего к неоднозначности.

Пример 1. Пусть в области определена функция по формуле . Аналитическим продолжением первоначальной функции будет функция , где в области функция . Аналитическим продолжением функции будет функция , где в области функция . Однако нельзя считать аналитическим продолжением первоначальной функции следующее выражение . Почему? Причина неприятная: выражение не является функцией от !!! Так как при значение вычисляется по формуле , но значение может принимать два значения, которые отличаются друг от друга на . Тогда принимает два ненулевых значения отличающихся друг от друга знаком. Получилась парадоксальная ситуация: начинали с голоморфной функции, а ее продолжение локально голоморфно (хорошие свойства исходной функции сохранило), но перестало быть функцией. Вроде бы сохранить голоморфность значительно труднее, чем остаться функцией. Как быть?

Б. Риман предложил «подправить» область определения так, чтобы устранить неоднозначность. Риман предложил точку считать двойной (наложенной друг на друга или полученной одну из другой в результате одного оборота вокруг начала координат); вторую (после одного оборота) точку считать лежащей на другом экземпляре комплексной плоскости, если значение до оборота отличается от значения после оборота, иначе считать лежащей на первоначальном экземпляре комплексной плоскости. Таким образом, у Римана область определения стала многолистной (лист – это экземпляр комплексной плоскости), до сих пор области определения функций были плоские или однолистные. Такие множества (новые области определения) оказываются являются гладкими многообразиями, которые широко применяются в современной математике. Области определения Римана – это первые (по хронологии) нетривиальные примеры многообразий в математике.

Пример 2. Риманова поверхность для .

Сохраним обозначения примера 1. В примере 1 введены три элемента . Пусть район действия (угол) первого элемента лежит в комплексной плоскости . Пусть район действия второго элемента лежит в комплексной плоскости . Пусть район действия третьего элемента лежит в комплексной плоскости . Поскольку значения и на пересечении совпадают, то согласно вышеуказанному принципу Римана точки отождествим (склеим) с точками . В результате склеивания общей части углов и получим угол . Итак, функция однозначная функция определена в угле . Поскольку значения и на пересечении совпадают, то согласно вышеуказанному принципу Римана точки отождествим (склеим) с точками . В результате склеивания общей части углов и получим «угол» , раствор которого больше и лежащий на двух экземплярах -плоскостей. Итак, функция однозначная функция определена в «угле» . Продолжаем склеивать районы действия элементов. Выберем следующий элемент

Поскольку значения и на пересечении совпадают, то согласно вышеуказанному принципу Римана точки отождествим (склеим) с точками . В результате склеивания общей части углов и получим «угол» , раствор которого больше и лежащий на трех экземплярах -плоскостей. Заметим, что значения при совпадают со значениями при , так как соответствующие отличаются ровно на .

Согласно вышеуказанному принципу Римана точки отождествим (склеим) с точками . В результате склеивания углов и получим риманову поверхность , так как дальнейшее склеивание не изменяет полученной поверхности.

Вопрос к экзамену: Принцип аргумента.

В дальнейшем начинаем знакомство с геометрической теорией функций комплексного переменного. Принцип аргумента играет большую роль не только для функций комплексного переменного. В нелинейном анализе есть понятие индекса вращения, которое является далеко идущим обобщением данного принципа. Из казахстанских математиков в своих исследованиях индексом вращения эффективно пользуется А. А.Женсыкбаев.

Задача 1. Решить уравнение в ограниченной подобласти области , где — односвязная область голоморфности функции .

Задача 2. (более скромная задача) Имеются ли решения уравнения в ограниченной подобласти области , где — односвязная область голоморфности функции ? Если имеются, то сколько?

Методы геометрической теории функций комплексного переменного дают на указанные вопросы ответ. Оказывается, достаточно посчитать некоторый интеграл. Его значение в точности совпадает с количеством нулей в функции . Больше того, вычисляемый интеграл имеет геометрическую трактовку.

Теорема 25. Интеграл совпадает с количеством нулей функциив подобласти .

Доказательство. Пусть — нуль функции порядка , причем . В окрестности точки справедливо представление (см. следствие 3 из теоремы 9)

Дифференцируя последнее равенство, имеем

Тогда отношение

В результате

Окончательно имеем

где — количество нулей функции в подобласти .

Теорема 26. Интеграл равен деленному на приращению аргумента функции при однократном обходе ориентированной границы подобласти .

Доказательство. Запишем уравнение границы в параметрическом виде . Тогда имеем

Теорема доказана.

Следствие из теорем 25 и 26. Количество нулей функциив подобласти совпадает с числом полных оборотов вокруг точки , которые сделает вектор , когда обходит границу .

Пример 1. Сколько нулей имеет многочлен в первом квадранте?

Решение. 1 шаг. Докажем, что вне круга многочлен не имеет

нулей. Это следует из оценки

2 шаг. В качестве подобласти возьмем сектор . Найдем приращение аргумента многочлена , когда обходит границу .

Вычисление

Вычисление

так как . Это следует из оценки

и того, что величина не выходит из круга . Поэтому вектор не может повернуться вокруг точки .

Вычисление

Выпишем мнимую и реальную части выражения в скобках.

При получаем . Следовательно, вначале точка находится в четвертом квадранте. Чтобы найти приращение аргумента проследим за движением точки , когда изменяется от до нуля. В следующей таблице приведены знаки значений

0

+

0

+

+

+

Точка

4 квадрант

Отр. мним. ось

3-квадрант

Отр. вещ. ось

2-квадрант

Пол. мним ось.

1-квадрант

Вращение по часовой стрелке – полный оборот

Таким образом, приращение при движении по мнимой оси от бесконечности до нуля равно .

Следовательно, . В первом квадранте многочлен не имеет нулей.

Лекция 13

Геометрические принципы ТФКП

Мы находимся в положении, несколько аналогичном положению человека, держащего в руках связку ключей и пытающегося открыть одну за другой несколько дверей. Рано или поздно ему всегда удается подобрать ключ к очередной двери, но сомнения относительно взаимно однозначного соответствия между ключами и дверями у него остаются.

Юджин Вигнер

Теорема Руше. Принцип сохранения области. Принцип максимума модуля. Лемма Шварца.

Вопрос к экзамену: Теорема Руше.

Задача 1. Решить уравнение в ограниченной подобласти области , где — односвязная область голоморфности функции .

Задача 2. (более скромная задача) Имеются ли решения уравнения в ограниченной подобласти области , где — односвязная область голоморфности функции ? Если имеются, то сколько?

Принцип аргумента дает решение задачи 2. Его можно модифицировать в следующем направлении. Вместо того чтобы искать приращение аргумента исходной функции можно находить приращение аргумента другой (более простой) функции. Единственное требование к выбору более простой функции заключается в том, чтобы их приращения были одинаковы. Руше указывает алгоритм нахождения простой функции с одинаковым приращением аргумента.

Теорема 27. (Руше) Если удается разложить на сумму , так что

    — голоморфная функция в , на границе области справедлива оценка или ,

то . Таким образом, вместо того чтобы решать задачу 2 для уравнения достаточно решить задачу 2 для уравнения .

Доказательство. Из неравенств

следует, что и не равны нулю на . Так как , то ,

то имеем соотношение

.

Но так как , то при любом изменении точка не выходит из круга . Поэтому вектор не может повернуться вокруг точки , то есть .Теорема доказана.

Пример 1. Где лежат нули функции ?

Решение. 1 шаг. Исследуем сначала единичный круг на наличие нулей исходного многочлена. В данном случае . Проверим выполнение условий теоремы Руше. Если , то верно неравенство . Следовательно, по теореме Руше уравнение в единичном круге имеет столько же решений, сколько решений имеет более простое уравнение . Ясно, что последнее уравнение в единичном круге не имеет решений. Итак, в единичном круге уравнение также не имеет решений.

2 шаг. Исследуем круг радиуса 2 и с центром в начале координат на наличие нулей исходного многочлена. Выберем

Проверим выполнение условий теоремы Руше. Если , то верно неравенство . Следовательно, по теореме Руше уравнение в круге имеет столько же решений, сколько решений имеет более простое уравнение . Ясно, что последнее уравнение в круге имеет пять решений (с учетом кратности). Итак, в круге уравнение также пять решений. В частности, вне круга многочлен не имеет нулей.

3 шаг. Итак, все нули исходного многочлена сосредоточены в кольце . Попробуем уточнить результат. Пусть

Проверим выполнение условий теоремы Руше. Если , то верно неравенство . Следовательно, по теореме Руше уравнение в круге имеет столько же решений, сколько решений имеет более простое уравнение . Ясно, что последнее уравнение в круге имеет пять решений (с учетом кратности). Итак, в круге уравнение также пять решений. В частности, вне круга многочлен не имеет нулей.

4 шаг. Итак, все нули исходного многочлена сосредоточены в кольце . Попробуем уточнить результат. Пусть

Проверим выполнение условий теоремы Руше. Если , то верно неравенство . Следовательно, по теореме Руше уравнение в круге имеет столько же решений, сколько решений имеет более простое уравнение . Ясно, что последнее уравнение в круге имеет пять решений (с учетом кратности). Итак, в круге уравнение также пять решений. В частности, вне круга многочлен не имеет нулей.

5 шаг. Итак, все нули исходного многочлена сосредоточены в кольце . Попробуем уточнить результат. Пусть

Проверим выполнение условий теоремы Руше. Если , то верно неравенство . Следовательно, условия теоремы Руше при таком выборе обозначений не выполняются. Надо выбирать по другому. В данном случае . Проверим выполнение условий теоремы Руше. Если , то верно неравенство . Следовательно, по теореме Руше уравнение в круге имеет столько же решений, сколько решений имеет более простое уравнение . Ясно, что последнее уравнение в круге не имеет решений. Итак, в круге уравнение также не имеет решений.

6 шаг. Итак, все нули исходного многочлена сосредоточены в кольце . Попробуем уточнить результат. В данном случае . Проверим выполнение условий теоремы Руше. Если , то верно неравенство . Следовательно, условия теоремы Руше при таком выборе обозначений не выполняются. Мой окончательный результат: все нули исходного многочлена сосредоточены в кольце .

Упражнение 25 для отличников. Улучшить результат лектора.

Упражнение 26. Любой многочлен степени имеет ровно нулей.

Вопрос к экзамену: Принцип сохранения области.

Часто функцию удобнее рассматривать как отображение области комплексной — плоскости на множество комплексной — плоскости. Если обладает достаточно хорошими свойствами, то определенные свойства наследуются множеством . К примеру,

·  внутренние точки переходят во внутренние (сохраняется внутренность),

·  внешние точки переходят во внешние (сохраняется внешность),

·  граничные точки переходят в граничные (соответствует границе граница).

Аккуратно сформулируем одно из таких утверждений.

Теорема 28. (Принцип сохранения области) Образ непостоянной голоморфной функции определенной на области представляет область.

Доказательство. Пусть — голоморфная функция отображает область на множество . Надо доказать, что — область, то есть

    — связное множество, — открытое множество.

Возьмем две произвольные точки из . Тогда найдутся такие, что . Поскольку — связное множество, то существует в непрерывная кривая , соединяющая с точкой . Поэтому образ кривой непрерывным образом соединяет с точкой . Связность доказана.

Возьмем произвольную точку из . Тогда найдется такая, что . Поскольку — открытое множество, то существует в существует круг . Поскольку нули функции изолированы[29], то круг можно выбрать так, что в обращается в нуль ровно один раз. На границе круга модуль является непрерывной функцией, поэтому из теоремы Вейерштрассе следует существование точки такой, что . Так как на границе функция не обращается в нуль, то . Покажем, что круг состоит из образов функции . Для любой точки из круга рассмотрим уравнение . Покажем, что это уравнение имеет решение. Применим теорему Руше для случая ( см. обозначения теоремы 27)

Проверим выполнение условий теоремы Руше

Поэтому уравнение в круге имеет столько же решений, сколько решений имеет уравнение в том же круге. Последнее уравнение имеет ровно одно решение. Тогда уравнение в круге также имеет решение. Теорема доказана.

Вопрос к экзамену: Принцип максимума модуля.

Нетривиальным следствием принципа сохранения области является принцип максимума модуля голоморфной функции.

Теорема 29. (Принцип максимума модуля) Модуль голоморфной в области функции достигает максимума во внутренней точке тогда и только тогда, когда функция – постоянна.

Доказательство. Из принципа сохранения области следует, что некоторая окрестность каждой внутренней точки отображается непостоянной голоморфной функцией в полную окрестность образа этой точки. Но в полной окрестности найдутся точки с модулями как меньше так и больше модуля образа рассматриваемой внутренней точки. Теорема доказана.

Упражнение 27. Для целой непостоянной функции введем величину . Доказать что монотонно возрастающая функция.

Вопрос к экзамену: Лемма Шварца.

Нетривиальным следствием принципа максимума модуля голоморфной функции является лемма Шварца.

Теорема 30. (Лемма Шварца) Если голоморфная функция отображает единичный круг в единичный круг, сохраняя центры, тогда для всех точек единичного круга верны неравенства . При этом если равенство достигается хотя бы в двух точках, то оно справедливо всюду, то есть в таком случае .

Доказательство. Функция имеет в точке устранимую особенность, так как и голоморфна в круге. Из принципа максимума следует, что

Отсюда имеем . Фиксируем и устремим к единице, тогда из последнего неравенства получим . Первая часть теоремы доказана. Если в разных двух точках достигается равенство , то по крайней мере одна из них отлична от нуля. Из принципа максимума следует, что достигает максимального значения равного единице. Тогда является постоянной функцией с модулем равным единице. Теорема доказана.

Лекция 14

Геометрические методы ТФКП

Подчинить вычисления своей воле, сгруппировать математические операции, научиться их классифицировать по степени трудности, а не по внешним признакам – вот это задачи математиков будущего, так, как я их понимаю, вот путь, по которому я хочу пойти…

Галуа

Принцип симметрии. Дробно — линейные отображения. Конформные изоморфизмы и автоморфизмы канонических областей.

Вопрос к экзамену: Принцип симметрии.

В данном пункте изучается вопрос, который можно было отнести к вопросу о способах аналитического продолжения функции. Но его геометрические возможности позволяют изучать этот вопрос здесь.

Теорема 31. (принцип симметрии) Пусть непересекающиеся области имеют общий прямолинейный участок границы , а функций соответственно голоморфны в и непрерывны на множествах . Тогда, если , то функция голморфна в области .

Доказательство. Для голоморфности функции по теореме Мореры достаточно проверить, что интеграл от нее по границе любого треугольника из равен нулю.

Если треугольник конечных размеров из и пересекает отрезок , то отрезком делится на две части и . Поэтому

Рассмотрим любую из частей и и для удобства обозначим ее снова через . Обозначим через трапецию, которая отсекается от прямой, параллельной и отстоящей от на расстоянии . Пусть

. По теореме Коши интеграл по равен нулю. Поэтому имеем равенство

. Пусть — основания трапеции . Так как длины боковых сторон стремятся к нулю при , а функция ограничена в замыкании , то . Ясно, что , но с разными ориентациями, поэтому при из последнего интегрального равенства имеем . Теорема доказана.

Вопрос к экзамену: Дробно — линейные отображения.

Голоморфная функция, у которой производная не обращается в нуль, порождает конформное отображение. Касательное к конформному отображению обладают свойствами

    сохраняет углы, квадраты преобразует в квадраты, окружности преобразует в окружности.

Глобальное конформное отображение должно быть еще и однозначным. В данном пункте изучим один класс конформных отображений.

Обозначим через множество всех отображений вида , где коэффициенты подчинены единственному ограничению . Последнее условие исключает постоянные функции. Перечислю свойства элементов из

    взаимно – однозначно отображает на , обратное к отображение также из , — конформное отображение, преобразует окружность в снова в окружность на , образует группу, если в качестве умножения рассматривать композицию, сохраняет симметрию, то есть образы симметричных точек симметричны, однозначно восстанавливается по образам трех различных точек, общий вид конформного отображения единичного круга на единичный круг имеет вид , общий вид конформного отображения верхней полуплоскости на единичный круг имеет вид ,

Доказательства приведенных утверждений носят геометрический характер и их наглядность зависит от соответствующих чертежей и рисунков. Опытному преподавателю на доказательство хватит пяти минут. Надо помнить, что в геометрических рассуждениях аналитические выкладки не играют решающей роли.

Вопрос к экзамену: Конформные изоморфизмы и автоморфизмы канонических областей.

Две односвязные области считаются конформно изоморфными, если существует конформное отображение одной области на другую. Когда эти области совпадают, то области называются автоморфными. Наша цель: классифицировать односвязные области, то есть определить все разные (неизоморфные между собой) классы односвязных областей. Каноническими областями назовем три области:

    расширенную комплексную плоскость , открытую комплексную плоскость , открытый единичный круг .

Теорема 32. Различные канонические области не могут быть изоморфными друг другу.

Доказательство. не изоморфно . От противного: пусть — конформное отображение. Тогда — ограниченная целая функция и по теореме Лиувилля – эта постоянная функция. Но тогда она не может иметь образ, совпадающий с . Противоречие.

не изоморфно . От противного: пусть — конформное отображение. Возьмем произвольную последовательность на , сходящуюся к бесконечности. Если последовательность ограниченна, то существует ее подпоследовательность сходящаяся к конечному числу. Тогда в силу непрерывности можно доопределить , положив равным пределу подпоследовательности. Но это число уже является значением в некоторой точке. Что противоречит однозначности . Если последовательность сходится к бесконечности, опять же получаем противоречие. Теорема доказана.

Теорема 33. Всякий конформный автоморфизм канонической области является ее дробно-линейным автоморфизмом.

Доказательство. Пусть — произвольный автоморфизм единичного круга . Обозначим образ центра через . Построим дробно-линейный автоморфизм . Композиция удовлетворяет всем условиям леммы Шварца, поэтому . Обратное отображение также удовлетворяет той же лемме.

. Таким образом, имеем . Отсюда .

Пусть — произвольный автоморфизм . Обозначим образ бесконечности через . Функция голоморфна всюду в , кроме точки , где она имеет полюс. Разложим в окрестности полюса в ряд Лорана . Построим функцию — это целая ограниченная функция, поэтому по теореме Лиувилля . Следовательно, имеем . Из однозначности следует, что — плюс первого порядка. Теорема доказана.

Автоморфимы имеют вид . Автоморфимы имеют вид .

Лекция 15

Теорема Римана

Геометрическое мышление всегда, по самому понятию геометрической фигуры, было абстрактным.

П. С.Александров

Принцип компактности. Теорема Гурвица. Теорема Римана.

Вопрос к экзамену: Принцип компактности.

Обычно в принципах компактности утверждается возможность выделения сходящейся подпоследовательности из бесконечного множества. То есть каким свойством должно обладать то или иное множество, чтобы гарантировалось существование сходящейся подпоследовательности. В случае числовой оси таким утверждением служит известная теорема Больцано-Вейерштрассе: из любой ограниченной бесконечной последовательности можно выделить сходящуюся подпоследовательность. В пространстве непрерывных на отрезке функций принцип компактности – это теорема Арцела-Асколи: из любой равномерно ограниченного и равностепенно непрерывного бесконечного множества непрерывных функций можно выделить равномерно сходящуюся подпоследовательность. В данном пункте обсуждается вопрос: каким свойством должно обладать семейство голоморфных функций, чтобы гарантировалось существование равномерно сходящейся подпоследовательности? Поскольку голоморфные функций являются также непрерывными, то остается справедливой теорема Арцела-Асколи. Оказывается, можно уточнить это утверждение в следующем направлении.

Теорема 34. (Принцип компактности) Из любой равномерно ограниченного внутри семейства голоморфных функций в области можно выделить равномерно сходящуюся на любом компакте из последовательность.

Схема доказательства (для троечников):

·  Лемма 1. Из любой последовательности, подчиненной условиям теоремы 34, можно выделить сходящуюся в каждой точке наперед заданного счетного, всюду плотного в множества.

·  Лемма 2. Равномерно ограниченное внутри семейство голоморфных функций равностепенно непрерывно внутри .

·  Лемма 3. Если последовательность, удовлетворяющая условиям теоремы 34, сходится в каждой точке всюду плотного в множества, то она равномерно сходится на каждом компакте из .

Доказательство леммы 1. Пусть — счетное, всюду плотное в множество. Пусть последовательность функций , удовлетворяющая условиям теоремы 34. Из числовой последовательности согласно теореме Больцано-Вейерштрассе выделим сходящуюся подпоследовательность. Это возможно, так как числовая последовательность — ограниченна. Обозначим выделенную подпоследовательность через . Из числовой последовательности согласно теореме Больцано-Вейерштрассе выделим сходящуюся подпоследовательность. Это возможно, так как числовая последовательность — ограниченна. Обозначим выделенную подпоследовательность через . Продолжаем бесконечно долго указанную процедуру выделения. В результате имеем

Отметим свойства строк данной таблицы:

    первая строка сходится в точке , вторая строка сходится в двух точках , третья строка сходится в трех точках и так далее, каждая последующая строка содержится в предыдущей строке.

Отметим свойство диагонали таблицы – диагональ сходится уже во всех точках , так как

    вся диагональ содержится в первой строке, содержится во второй строке, содержится в третьей строке так далее.

Лемма 1 доказана.

Доказательство леммы 2. Возьмем произвольный круг из . Для всех справедлива оценка произвольного элемента данного семейства :

Константа не зависит от конкретного элемента семейства , то есть пригодна для всего семейства. Поэтому семейство равностепенно непрерывно в произвольном круге из . Поскольку произвольный компакт из по определению может быть покрыт конечным набором кругов вида из , то семейство равностепенно непрерывно на произвольном компакте из . Лемма 2 доказана.

Доказательство леммы 3. Если последовательность из семейства сходится в каждой точке всюду плотного в множества , то она сходится равномерно на каждом компакте из . Докажем это. Фиксируем и компакт из . Из равностепенной непрерывности семейства следует существование такого, что при любых справедливо неравенство . Выберем разбиение на квадраты с диагоналями меньшими , тогда для любых точек , принадлежащих одному квадрату, и любой функции справедливо неравенство . Компакт покрывается конечным числом выбранных квадратов. Так как плотно в , то в каждом квадрате найдется хотя бы одна точка из . Пусть

По критерию Коши отсюда следует утверждение леммы 3. Принцип компактности доказан П. Монтелем.

Вопрос к экзамену: Теорема Гурвица.

Теорема 35. (Гурвиц) Пусть последовательность , голоморфных в области , равномерно на любом компакте из сходится к непостоянной функции . Тогда, если , то в любом круге все функции (при некотором ) обращаются в нуль.

Доказательство. Так как последовательность функции сходится равномерно к функции , то интеграл по произвольному треугольнику функции есть предел интегралов по тем же треугольникам последовательности . Такой предел равен нулю. Тогда по теореме Мореры предельная функция голоморфна на каждом компакте. Ясно, что ее нули изолированы. Возьмем проколотую окрестность ее нуля, где предельная функция не обращается в нуль. Найдем минимум модуля предельной функции на границе этой (уже не проколотой) окрестности. Ясно, что этот минимум отличен от нуля. Обозначим его через . Выберем номер так, чтобы выполнялось

Тогда по теореме Руше уравнения и в указанной выше окрестности имеют одинаковое число нулей. То есть имеет нули в указанной окрестности. Теорема доказана.

Следствие из теоремы 35. Если последовательность однозначных и голоморфных в области функций сходится равномерно на каждом компакте из , то предельная функция этой последовательности либо постоянна, либо однозначна.

Вопрос к экзамену: Теорема Римана.

Две односвязные области считаются конформно изоморфными, если существует конформное отображение одной области на другую. Когда эти области совпадают, то области называются автоморфными. Наша цель: классифицировать односвязные области, то есть определить все разные (неизоморфные между собой) классы односвязных областей. Каноническими областями назовем три области:

    расширенную комплексную плоскость , открытую комплексную плоскость , открытый единичный круг .

Теорема 32. Различные канонические области не могут быть изоморфными друг другу.

В данном пункте докажем, что среди односвязных областей нет неизоморфных каноническим областям. То есть каждая односвязная область посредством конформного отображения может быть переведена либо в круг, либо на открытую плоскость, либо на расширенную плоскость. Это очень важный результат. К примеру профиль крыла самолета можно конформно отобразить на профиль круга и редуцировать задачи, возникшие для крыла, к задачам для круга.

Теорема 36. (Б. Риман) Любая односвязная область, граница которой содержит более одной точки, изоморфна единичному кругу.

Схема доказательства (для троечников):

    сначала покажем, что данную область можно конформно отобразить в круг, затем покажем, что на множестве всех конформных отображений данной области в круг достигается максимум некоторого функционала, проверим, что конформное отображение, на котором достигается максимум предыдущего функционала – есть требуемое отображение.

Доказательство. 1 шаг. Пусть область — односвязная область, граница которой содержит по крайней мере две точки. Итак, . Можно считать, что одна из этих точек бесконечно удаленная точка, иначе дробно-линейное отображение позволит получить новую область , у которой на границе содержится бесконечность. Итак, граница содержит разрез от нуля до бесконечности. Тогда по теореме о монодромии в области можно выделить однозначную ветвь функции . В результате область конформно отобразится в новую область , которая обязательно имеет внешнюю часть. То есть существует круг , который не имеет общих точек ни с , ни с . Это следует из того, что другая ветвь функции не обращается в нуль на и ее образ не имеет общих точек с . Круг надо выбирать из . Все предыдущие рассуждения можно не делать, если Вы с самого начала можете выбрать внешний по отношению к исходной области круг . Введем голоморфную в функцию по формуле , модуль которой меньше единицы. Следовательно, образ лежит внутри единичного круга. Таким образом, исходную область всегда можно конформно отобразить в единичный круг.

2 шаг. Обозначим через семейство всех голоморфных и однозначных в функций, по модулю ограниченных единицей. Это непустое семейство (смотри 1 шаг). Это равномерно ограниченное семейство, поэтому из теоремы Монтеля следует его компактность. Возьмем произвольную точку из единичного круга из условия и составим функционал . Часть семейства , состоящая из функций , для которых , компактна в себе. То есть это множество содержит все равномерные пределы сходящихся последовательностей из . Заметим, что равномерный предел указанных последовательностей будет однозначной функцией, так как . Покажем, что максимум функционала на множестве достигается. Обозначим через . Тогда найдется последовательность такая, что . Так как компактно в себе, то существует равномерно сходящаяся к некоторому элементу из подпоследовательность .Из неравенств следует, что максимум функционала на множестве достигается.

3 шаг. Проверим, что предельная функция конформно отображает на круг.

    функция — однозначная и голоморфная функция, , иначе модуль производной функции в точке больше модуля производной функции в той же точке. функция отображает на круг. В противном случае найдется тока , у которой нет — прообразов. Тогда модуль производной функции в точке больше модуля производной функции в той же точке.

Теорема доказана.

Лекция 16

Некоторые применения

Ряд чисел мыслится уже неограниченно продолжаемым, и с ним в математику вступает бесконечность

А. Д.Александров

Эллиптический синус. Модулярная функция. Формула Кристоффеля-Шварца.

Вопрос к экзамену: Эллиптический синус.

Заметим, что непрерывные периодические функций на числовой оси имели наименьший период. К примеру, наименьший период тригонометрического синуса равен . А нельзя ли придумать непрерывную периодическую функцию с несколькими независимыми (несоизмеримыми) периодами?

Упражнение 28. Непостоянная непрерывная периодическая функция не может иметь несколько несоизмеримых периодов.

Это упражнение касается функций определенных на числовой оси. Совсем иначе обстоит дело с функциями определенными на плоскости. Приведем пример функции с двумя независимыми периодами. Дальнейшее обобщение подобных понятий приводит к теории автоморфных функций. Функция является автоморфной по отношению к некоторой группе, если она принимает одни и те же значения в конгруэнтных точках.

Итак, надо придумать двояко периодическую функцию. Как же вводились периодические на оси функций? Я напомню следующий способ. Алгоритм введения тригонометрического синуса

1 шаг. Вычислим интеграл с верхним переменным пределом . Обозначим результат

2 шаг. Найдем обратную к полученной функции. Очевидно, что получим тригонометрический синус .

Указанный способ введения периодических функций обобщается. К примеру, юный Гаусс (1796, 19 лет – ровесник студента 3 курс мехмата) заинтересовался интегралом и первым открыл эллиптические функций и их важнейшие свойства. Сегодня мы рассмотрим интеграл при .

Упражнение 29. Найдем образ прямой при отображении

Решение. Пусть , то . Это начальная точка. Допустим, что растет, начиная с нуля. Тогда, считая подынтегральное выражение положительным, видим, что начинает вместе с возрастать . Это происходит до тех пор пока значение не достигнет единицы. В результате также возрастет от нуля до величины . При переходе через точку один из четырех линейных множителей подкоренного выражения, в частности , меняет знак. Будем рассматривать однозначную ветвь корня в верхней полуплоскости . Это соответствует обходу точки по малой полуокружности . В результате такого обхода меняется от до , а аргументы остальных множителей не получают приращений. Поэтому на отрезке оси аргумент корня равен , а аргумент подынтегрального выражения равен . Значения на отрезке можно представить в виде

Заметим, что интеграл принимает только положительные значения при . Когда описывает слева направо отрезок оси , точка описывает снизу вверх отрезок плоскости , где . При переходе через точку еще один множитель подкоренного выражения меняет знак. Как и выше, убедимся в том, что рассматриваемая ветвь корня на луче оси должна иметь аргумент , то есть принимать отрицательные значения. Значения на этом луче представляются в виде

Последний интеграл принимает положительные значения. Замена переменных показывает, что . Поэтому луч функция преобразует в отрезок плоскости . Точно также исследуется образ левой полуоси .

Упражнение 30. Верхнюю полуплоскость функция преобразует в замкнутый прямоугольник.

Решение. Применим принцип соответствия границ. Согласно упражнению 29 ось преобразуется в границу прямоугольника с вершинами . Тогда внутренность преобразуется во внутренность соответствующего прямоугольника.

Упражнение 31. Пусть обращение интеграла , отображающее прямоугольник на верхнюю полуплоскость. Тогда аналитическое продолжение на всю плоскость есть двояко периодическая функция, то есть .

Решение. Применяем принцип симметрии. Правда, при таком продолжении получается мероморфная функция. То есть продолжение имеет особые точки в виде простых полюсов. Как говорит П. Монтель[17]: «Мероморфные функций должны рассматриваться как голоморфные в расширенной комплексной плоскости». Поэтому ничего страшного в этом нет. Полученная функция называется эллиптическим синусом.

Вопрос к экзамену: Модулярная функция.

Модулярная группа преобразований состоит из дробно-линейных отображений причем . Модулярной функцией называется функция, которая принимает одни и те же значения в точках . Короче говоря, это автоморфная по отношению к модулярной группе функция. Иначе говоря, мы хотим привести еще одно нетривиальное применение геометрических методов теории функций комплексного переменного.

Рассмотрим круговой треугольник , образованный дугами окружностей ортогональных к единичной окружности.

По теореме Римана существует конформное отображение этого треугольника на верхнюю полуплоскость. Причем можно добиться того, чтобы . По принципу симметрии функцию можно аналитически продолжить в симметричные относительно сторон исходного треугольника новые треугольники. Продолжим продолжение бесконечно долго. В результате получим голоморфную в единичном круге функцию , которая называется модулярной функцией. Из построения ясно, что модулярная функция внутри окружности не принимает три значения . Теперь опишем обратную к модулярной функции аналитическую функцию. Рассмотрим ее однозначную ветвь голоморфную в верхней полуплоскости и отображающую эту полуплоскость на треугольник . Полученную функцию по принципу симметрии можно продолжить. Продолженная функция – бесконечнозначна, точки являются ее логарифмическими точками ветвления, все значения лежат в единичном круге.

Следствие из того, что существует обратная к модулярной. (Малая теорема Пикара) Любая целая функция, отличная от постоянной, принимает все (конечные) комплексные значения, за исключением, быть может, одного.

Доказательство. Пусть целая функция не принимает двух различных значений . Функция также целая и не принимает значений . В окрестности произвольной точки голоморфна функция , где — какая-либо ветвь функции, обратной к модулярной, голоморфная в окрестности точки . Так как функция не принимает значений — особых для аналитической функции , то функция продолжаема вдоль любого пути на комплексной плоскости. По теореме о монодромии функция однозначна и голоморфна на всей комплексной плоскости. Причем все ее значения лежат в единичном круге. Тогда по теореме Лиувилля функция — постоянна. Что противоречит непостоянству .

Пример. Функция

Исключительные значения целых функций могут быть, а иногда исключительных значений нет.

Вопрос к экзамену: Формула Кристоффеля-Шварца.

В этой лекции излагаются различные приложения ТФКП. К примеру, теорема Римана – это теоретический результат: она дает уверенность существования некоторого конформного отображения, но не дает способа конструктивного построения этого отображения. На практике очень важен конкретный вид конформного отображения.

Задача. (Кристоффель, Шварц) Найти формулу конформного отображения произвольного многоугольника на полуплоскость или наоборот: полуплоскости на многоугольник.

Обсуждение.

·  Существование такого отображения следует из теоремы Римана. Так как верхняя полуплоскость и внутренность многоугольника по теореме Римана могут быть конформно отображены в единичный круг, то достаточно использовать принцип суперпозиции.

·  Мы уже решали задачу отображения полуплоскости на прямоугольник (смотри – эллиптический синус). Вспомним ответ:

·  Попробуем преобразовать последний интеграл в виде

и интерпретировать полученные множители следующим образом:

1.  прямоугольник имеет четыре угла, поэтому под интегралом четыре множителя;

2.  каждый угол в прямоугольнике равен , поэтому показатели скобок равны ,

3.  точки являются прообразами вершин прямоугольника (смотри – эллиптический синус), поэтому подынтегральные скобки обращаются в нуль в указанных точках.

·  Ожидается, что такого же вида интегралы будут осуществлять конформные отображения полуплоскости на многоугольник.

Теперь мы готовы, по крайней мере, морально к выводу формулы Кристоффеля-Шварца.

Обозначим через — последовательные вершины многоугольника , — угол многоугольника в вершине , — прообраз вершины при отображении .

Теорема 37. Пусть функция конформно отображает полуплоскость на ограниченный многоугольник Тогда имеет место формула Кристоффеля-Шварца:

где — постоянные и интеграл берется по кривой, лежащей в полуплоскости .

Схема доказательства.

К) Пусть — полная аналитическая функция, полученная продолжением функции определенной на верхней полуплоскости. Вообще говоря, эта многозначная функция. Однако из принципа симметрии следует, что отношение

— однозначная и голоморфная функция на всей комплексной плоскости с выколотыми точками .

А) В окрестности бесконечно удаленной точки функция имеет асимптотическое представление

Н) В окрестности точки функция имеет асимптотическое представление

где функция голоморфна в точке и .

Г) В окрестности точки функция имеет асимптотическое представление

где функция голоморфна в окрестности точки и .

У) Надо показать представление

или

Ж) Остается решить дифференциальное уравнение

ЧАСТЬ 2

ПРАКТИЧЕСКИЕ ЗАНЯТИЯ

ПО ТЕОРИИ ФУНКЦИЙ

КОМПЛЕКСНОГО ПЕРЕМЕННОГО

Занятие № 1

Тема: Комплексные числа

Нумерация примеров соответствует книге [ 1 ].

Пример 1.1. Выполните следующую операцию над комплексными числами: .

Решение. Матричный способ. На лекции утверждалось, что каждому действительному числу ставится в соответствие диагональная матрица . Тогда единица представляется в виде матрицы , а мнимая единица – в виде матрицы . Поэтому имеем матричное соотношение

Решение. Способ, использующий каноническую запись комплексного числа. На лекции утверждалось, что каждое комплексное число может быть записано в виде . Поэтому , так как в полярной системе координат именно такие соответствуют векторам . Тогда справедливы равенства

Пример 1.2. Выполните следующую операцию над комплексными числами: .

Решение. Геометрический способ. На комплексной плоскости отметим точки и , затем вспомним геометрический смысл операции деления:

    поделим модули чисел и друг на друга, в результате получим единицу вектор повернем против часовой стрелки на угол

 

Преподавателю надо обосновать,

полученный ответ по рисунку.

Пример 1.3. Выполните следующую операцию над комплексными числами: .

Решение. Использование алгебраической формы записи комплексных чисел. На лекции утверждалось, что каждое комплексное число имеет сопряженное число . Тогда справедливы равенства

Пример 1.4. Выполните следующую операцию над комплексными числами: .

Решение. Матричный способ.

Решение. Геометрический способ.

На рисунке на окружности радиуса 2 отмечена точка . Чтобы возвести это число в куб, надо повернуть два раза соответствующий радиус против часовой стрелки на угол и удлинить радиус до восьми. Получим -8.

Пример 2.1. Найдите модуль и аргумент комплексного числа : .

Решение. Комплексное число соответствует вектору , поэтому полярный угол равен , а радиус равен . Кратко запишем .

Пример 2.2. Найдите модуль и аргумент комплексного числа : .

Решение. Комплексное число соответствует вектору , поэтому полярный угол равен , а радиус равен . Кратко запишем .

Пример 2.3. Найдите модуль и аргумент комплексного числа : .

Решение. Комплексное число соответствует вектору , поэтому полярный угол равен , а радиус равен . Кратко запишем .

Пример 2.4. Найдите модуль и аргумент комплексного числа : .

Решение. Комплексное число соответствует вектору , поэтому полярный угол равен , а радиус равен . Кратко запишем .

Пример 2.5. Найдите модуль и аргумент комплексного числа : .

Решение. Комплексное число соответствует вектору , поэтому полярный угол равен , а радиус равен . Кратко запишем

Пример 2.6. Найдите модуль и аргумент комплексного числа : .

Решение. Комплексное число соответствует вектору , поэтому полярный угол равен , а радиус равен . Кратко запишем .

. Пример 2.7. Найдите модуль и аргумент комплексного числа : .

Решение. Комплексное число соответствует вектору , поэтому полярный угол равен , а радиус равен . Кратко запишем

Пример 2.8. Найдите модуль и аргумент комплексного числа : .

Решение. Комплексное число соответствует вектору , поэтому полярный угол равен , а радиус равен . Кратко запишем

Пример 4.1. Найдите корни из комплексного числа и покажите их на чертеже: .

Решение. Обозначим искомый корень через . Тогда . Напомним, что равенство двух комплексных чисел влечет равенство их модулей и аргументов. Поэтому из следует два равенства: . Следовательно, имеем соотношения . В частности, можем записать

В результате получаем три различных корня

Вывод: корни расположены на окружности и делят ее на равные дуги. Поэтому достаточно отметить один корень , а затем разделить окружность на три равные части так, чтобы одна из точек деления совпадала с .

Пример 4.2. Найдите корни из комплексного числа и покажите их на чертеже: .

Решение. Обозначим искомый корень через . Тогда . Напомним, что равенство двух комплексных чисел влечет равенство их модулей и аргументов. Поэтому из следует два равенства: . Следовательно, имеем соотношения . В частности, можем записать

В результате получаем три различных корня

Вывод: корни расположены на окружности и делят ее на равные дуги. Надо начинать с корня и затем разделить окружность на три дуги.

Пример 4.3. Найдите корни из комплексного числа и покажите их на чертеже: .

Решение. Вспомните выводы примеров 4.1 и 4.2. Для того чтобы записать ответ данной задачи надо:

знать радиус окружности, отметить на ней один из корней, разделить окружность на равные дуги.

Внимательный анализ решений предыдущих примеров позволяет заключить, что:

радиус равен 1, , надо делить на четыре части.

На рисунке приведена полная информация. Остается записать правильный ответ. Желаю удачи!

Пример 4.4. Найдите корни из комплексного числа и покажите их на чертеже: .

Решение. Вспомните выводы примеров 4.1 и 4.2. Для того чтобы записать ответ данной задачи надо:

знать радиус окружности, отметить на ней один из корней, разделить окружность на равные дуги.

Внимательный анализ решений предыдущих примеров позволяет заключить, что:

радиус равен , надо делить на шесть частей.

Запишите ответ и сравните его с книжным ответом.

Пример 4.5. Найдите корни из комплексного числа и покажите их на чертеже: .

Решение приведено на рисунке (без комментариев)

 

Пример 4.6. Найдите корни из комплексного числа и покажите их на чертеже: .

Решение приведено на рисунке (без комментариев)

 

Пример 4.7. Найдите корни из комплексного числа и покажите их на чертеже: .

Решение приведено (без комментариев)

Пример 23.1. Геометрически интерпретировать следующее соотношение: .

Решение. Вспомним, что модуль разности двух комплексных чисел геометрически представляет расстояние между этими точками. Комплексное число – это точка на плоскости. Тогда неравенство означает, что расстояние от текущей точки до фиксированной точки меньше . Множество таких образует круг с центром в и радиуса .

Пример 23.2. Геометрически интерпретировать следующее соотношение: .

Решение. Вспомним, что модуль разности двух комплексных чисел геометрически представляет расстояние между этими точками. Комплексное число – это точка на плоскости. Тогда неравенство означает, что расстояние от текущей точки до фиксированной точки равно . Множество таких образует окружность с центром в и радиуса .

Пример 23.3. Геометрически интерпретировать следующее соотношение: .

Решение. Вспомним, что модуль разности двух комплексных чисел геометрически представляет расстояние между этими точками. Комплексное число – это точка на плоскости. Тогда неравенство означает, что расстояние от текущей точки до фиксированной точки меньше . Множество таких образует внешность круга с центром в и радиуса .

Пример 24. Геометрически интерпретировать следующее соотношение: .

Решение. Из решения примера 23 следует, что условие примера 24 можно интерпретировать как

Это эллипс (по определению).

Пример 25. Геометрически интерпретировать следующее соотношение:

Решение. Из решения примера 23 следует, что условие примера 25 можно интерпретировать как

Это «внешность» гиперболы (по определению). Для полноты изложения аккуратно запишем уравнение соответствующей гиперболы в каноническом виде. Пусть .

Тогда

Асимптоты имеют вид . Остается предъявить рисунок.

Подпись:

Пример 26. Геометрически интерпретировать следующее соотношение: .

Решение. Здесь фиксированные точки. Надо указать всевозможные точки , отстоящие от данных точек на одинаковых расстояниях. Это по определению прямая перпендикулярная к отрезку и проходящая через середину этого отрезка.

Пример 27.1. Геометрически интерпретировать следующее соотношение:

Решение. Какие имеют реальные части равные константе ? Это вертикальная прямая, проходящая через точку . Тогда означает правую полуплоскость с границей .

Пример 27.2. Геометрически интерпретировать следующее соотношение: .

Решение. означает нижнюю полуплоскость с границей , где — горизонтальная прямая.

Пример 28. Геометрически интерпретировать следующее соотношение: .

Решение. Заметим, что умножение мнимую единицу соответствует повороту на 90 градусов против часовой стрелки вектора . (Как из получить ?) Поэтому сначала начертим область , а затем эту область повернем на 90 градусов по часовой стрелке. Область есть объединение областей . Каждая из этих областей рассмотрена в примере 27. Следовательно, представляет вертикальную полосу с границей . Поворот этой полосы на 90 градусов по часовой стрелке приведет к горизонтальной полосе, ограниченной прямыми .

Пример 29. Геометрически интерпретировать следующее соотношение:

Решение. Множество , у которых представляет луч соответствующий углу раствора . Множество , у которых представляет луч соответствующий углу раствора . Тогда представляет угол раствора . Надо начертить соответствующий чертеж.

Пример 30. Геометрически интерпретировать следующее соотношение: .

Решение. Пусть , тогда . Получилась парабола с горизонтальной осью симметрии, причем ее ветви направлены направо. Надо начертить соответствующий чертеж.

Пример 61.1. Найдите модуль и аргумент комплексного числа : .

Решение. Так как , то . Вывод: модуль экспоненты определяется реальной частью ее показателя, а аргумент экспоненты определяется мнимой частью ее показателя.

Пример 61.2.Найдите модуль и аргумент комплексного числа : .

Решение. Так как

то . Поэтому .

Пример 61.2.Найдите модуль и аргумент комплексного числа : .

Решение. Так как , то

Пример 61.3.Найдите модуль и аргумент комплексного числа : .

Решение. Так как , то

Пример 61.4.Найдите модуль и аргумент комплексного числа : .

Решение. Так как , то

Вопрос для вдумчивого читателя: чему равен ? Надо знать разницу между и .

Литература

1 Волковыский Л. И., Лунц Г. Л.,Араманович И. Г. Сборник задач по теории функций комплексного переменного — М.:Наука. 1975. 320 с.

Занятие № 2

Тема: Элементарные функции

Нумерация примеров соответствует книге [ 1 ].

Пример 62.1. Вычислить сумму тригонометрических функций: .

Решение. Поскольку , то

Пример 62.2. Вычислить сумму тригонометрических функций: .

Решение. Поскольку , то

Пример 71.1. Вычислить все значения логарифма: .

Решение. Обозначим искомый корень через . Тогда

Пример 71.2. Вычислить все значения логарифма: .

Решение. Обозначим искомый корень через . Тогда

Пример 71.3. Вычислить все значения логарифма: .

Решение. Обозначим искомый корень через . Тогда

Пример 71.4. Вычислить все значения логарифма: .

Решение. Обозначим искомый корень через . Тогда

Пример 71.5. Вычислить все значения логарифма: .

Решение. Обозначим искомый корень через . Тогда

Пример 116. Найдите образы координатных линии при отображении .

Решение. Если , то , где произвольное действительное число. Следовательно, . Отсюда получаем , которое представляет параметрическое уравнение некоторой кривой. Роль параметра играет . Найдем явное уравнение кривой, исключая параметр. В результате имеем . Таким образом, на комплексной плоскости получаем параболу .

Если , то , где произвольное действительное число. Следовательно, . Отсюда получаем , которое представляет параметрическое уравнение некоторой кривой. Роль параметра играет . Найдем явное уравнение кривой, исключая параметр. В результате имеем . Таким образом, на комплексной плоскости получаем параболу. Подробности на рисунке.

Пример 117. Найдите образы координатных линии при отображении .

Решение. Если , то , где произвольное действительное число. Следовательно, . Отсюда получаем, которое представляет параметрическое уравнение некоторой кривой. Роль параметра играет . Найдем явное уравнение кривой, исключая параметр. В результате имеем . Таким образом, на комплексной плоскости получаем окружность.

Если , то , где произвольное действительное число. Следовательно, . Отсюда получаем, которое представляет параметрическое уравнение некоторой кривой. Роль параметра играет . Найдем явное уравнение кривой, исключая параметр. В результате имеем . Таким образом, на комплексной плоскости получаем окружность (вышеприведенный рисунок надо повернуть на 90 градусов).

Литература

1 Волковыский Л. И., Лунц Г. Л.,Араманович И. Г. Сборник задач по теории функций комплексного переменного — М.:Наука. 1975. 320 с.

Занятие № 3

Тема: Условия Коши-Римана

Нумерация примеров соответствует книге [ 1 ].

Пример 132. При каких значениях параметров функция будет голоморфной?

Решение. На лекции утверждалось, что частные производные реальной и мнимой частей голоморфных функций обязаны удовлетворять условиям Коши-Римана. Поэтому сначала выделим реальную и мнимую части данной функций, а затем вычислим их частные производные и выберем параметры так, чтобы выполнились условия Коши-Римана. 1 шаг. Нахождение .

2 шаг. Вычисление производных.

3 шаг. Выбор параметров. Запишем условия Коши-Римана

Поэтому будем выбирать параметры согласно системе

Отсюда имеем . Таким образом, исключая параметр из определения функции , получаем

4 шаг. Проверка голоморфности. Надо убедиться, что существует .

Пример 159. Зная реальную часть голоморфной функции, надо восстановить голоморфную функцию.

Решение. Так как , то достаточно найти ее мнимую часть. Из условии Коши-Римана следует, что можно считать известными частные производные мнимой части. Таким образом, надо по частным производным найти неизвестную функцию. Это несложная задача и ее решение приведено ниже.

1 шаг. Нахождение частных производных .

2 шаг. Вычисление мнимой части по формуле

, ее многие знают из курса математического анализа. Точку в данном случае можно выбирать по своему усмотрению. Пусть . Еще один важный момент. Форма контура интегрирования также зависит от Вашего настроения. Контур показан жирной ломанной линией на рисунке. Ломанная линия состоит из двух отрезков, поэтому

3 шаг. Построение искомой функции.

4 шаг. Проверка правильности ответа, то есть надо убедиться, что

Вывод: голоморфная функция по реальной части восстанавливается с точностью до константы.

Замечание для доброжелательных читателей. Реальная часть голоморфной функции удовлетворяет уравнению Лапласа, то есть ее нельзя задавать произвольно.

Пример 160. Зная реальную часть голоморфной функции, надо восстановить голоморфную функцию.

Решение. 1 шаг. Нахождение частных производных .

2 шаг. Вычисление мнимой части по формуле

, ее многие знают из курса математического

анализа. Предостережение: контур интегрирования не должен проходить через точку . Тогда

Заглянув в ответы Демидовича Б. П. [ 2 ], можем записать .

3 шаг. Построение искомой функции.

4 шаг. Проверка правильности ответа, то есть надо убедиться, что

Литература

1 Волковыский Л. И., Лунц Г. Л.,Араманович И. Г. Сборник задач по теории функций комплексного переменного — М.:Наука. 1975. 320 с.

Занятие № 4

Тема: Геометрический смысл модуля и аргумента производной функции

Нумерация примеров соответствует книге [ 1 ].

Пример 187.1. Выяснить геометрический смысл модуля и аргумента производной следующей функции при .

Решение. Найдем производную в точке :

Запишем приращение функции в окрестности точки

Обозначим через значение функции . Тогда последнее равенство примет вид

Требуемый геометрический смысл вытекает из соотношения .

·  Если пробегает по окружности достаточно малого радиуса, то из предыдущей формулы вытекает, что пробегает некоторую кривую на плоскости «близкую» к окружности . То есть происходит растяжение в три раза – это за счет модуля производной .

·  Применяя к обеим частям операцию , получаем равенство . Следовательно, приращение почти совпадает с приращением . Это связано с тем, что . То есть происходит поворот на ноль радиан.

Пример 187.2. Выяснить геометрический смысл модуля и аргумента производной следующей функции при .

Решение. Найдем производную в точке :

Запишем приращение функции в окрестности точки

Обозначим через значение функции . Тогда последнее равенство примет вид

Требуемый геометрический смысл вытекает из соотношения .

·  Если пробегает по окружности достаточно малого радиуса, то из предыдущей формулы вытекает, что пробегает некоторую кривую на плоскости «близкую» к окружности . То есть происходит сжатие в раз – это за счет модуля производной .

·  Применяя к обеим частям операцию , получаем равенство . Следовательно, приращение почти совпадает с приращением . Это связано с тем, что . То есть происходит поворот на ноль радиан.

Пример 187.3. Выяснить геометрический смысл модуля и аргумента производной следующей функции при .

Решение. Найдем производную в точке :

Запишем приращение функции в окрестности точки

Обозначим через значение функции . Тогда последнее равенство примет вид

Требуемый геометрический смысл вытекает из соотношения .

·  Если пробегает по окружности достаточно малого радиуса, то из предыдущей формулы вытекает, что пробегает некоторую кривую на плоскости «близкую» к окружности . То есть происходит растяжение в 6 раз – это за счет модуля производной .

·  Применяя к обеим частям операцию , получаем равенство . Следовательно, приращение получается из приращения прибавлением постоянного угла. Это связано с тем, что . То есть происходит поворот на 90 градусов против часовой стрелки вокруг точки .

Пример 187.4. Выяснить геометрический смысл модуля и аргумента производной следующей функции при .

Решение. . Ответ: модуль производной в окрестности соответствующей точки растягивает в 75 раз, аргумент производной вращает на угол против часовой стрелки.

Литература

1 Волковыский Л. И., Лунц Г. Л.,Араманович И. Г. Сборник задач по теории функций комплексного переменного — М.:Наука. 1975. 320 с.

Занятие № 5

Тема: Интегрирование функций комплексного переменного

Нумерация примеров соответствует книге [ 1 ].

Пример 386. Вычислить интеграл , когда контур интегрирования либо отрезок, либо ломанная.

Решение. Сначала вычислим интеграл вдоль отрезка, соединяющего концевые точки интеграла. Затем найдем значение интеграла вдоль катетов (смотри рисунок).

1 шаг. Уравнение контура интегрирования. Уравнение отрезка : . Последнее эквивалентно .

2 шаг. Сведение к определенным интегралам. Тогда исходный интеграл сведем к комбинаций определенных интегралов.

Теперь проведем вычисления вдоль катетов.

Уравнение горизонтального катета имеет вид и соответствующий интеграл . Уравнение вертикального катета имеет вид и соответствующий интеграл . Тогда исходный интеграл равен сумме найденных интегралов.

Гипотеза: ответы одинаковы, то есть интеграл не зависит от формы кривой, соединяющая концевые точки.

Пример 393.1. Вычислить интеграл , когда контур интегрирования – полуокружность с началом и обход производится против часовой стрелки. Непрерывная функция выбрана так, что .

Решение. Поскольку , то справедливо представление . Тогда непрерывная ветвь , принимающая значение , может быть записана в виде функций . Теперь готовы вычислить исходный интеграл

Пример 393.2. Вычислить интеграл , когда контур интегрирования – полуокружность с началом и обход производится против часовой стрелки. Непрерывная функция выбрана так, что .

Решение. Непрерывная ветвь , принимающая значение , может быть записана в виде функций . Ответ отличается от ответа предыдущего примера только знаком: 2-2i.

Пример 393.3. Вычислить интеграл , когда контур интегрирования – полуокружность с началом и обход производится против часовой стрелки. Непрерывная функция выбрана так, что .

Решение. Поскольку , то справедливо представление . Тогда непрерывная ветвь , принимающая значение , может быть записана в виде функций . Теперь готовы вычислить исходный интеграл

Пример 393.4. Вычислить интеграл , когда контур интегрирования –окружность с началом и обход производится против часовой стрелки. Непрерывная функция выбрана так, что .

Решение. Поскольку , то справедливо представление . Тогда непрерывная ветвь , принимающая значение , может быть записана в виде функций . Теперь готовы вычислить исходный интеграл

Пример 393.4. Вычислить интеграл , когда контур интегрирования –окружность с началом и обход производится против часовой стрелки. Непрерывная функция выбрана так, что .

Решение. Поскольку , то справедливо представление . Тогда непрерывная ветвь , принимающая значение , может быть записана в виде функций . Теперь готовы вычислить исходный интеграл

Пример 394.1. Вычислить интеграл , когда контур интегрирования –окружность с началом и обход производится против часовой стрелки. Непрерывная функция выбрана так, что .

Решение. Поскольку , то справедливо представление . Тогда непрерывная ветвь , принимающая значение , может быть записана в виде функций . Теперь готовы вычислить исходный интеграл

Пример 394.2. Вычислить интеграл , когда контур интегрирования –окружность с началом и обход производится против часовой стрелки. Непрерывная функция выбрана так, что .

Решение. Поскольку , то справедливо представление . Тогда непрерывная ветвь , принимающая значение , может быть записана в виде функций . Теперь готовы вычислить исходный интеграл

Пример 394.3. Вычислить интеграл , когда контур интегрирования –

окружность с началом и обход производится против часовой стрелки. Непрерывная функция выбрана так, что .

Решение. Поскольку , то справедливо представление . Тогда непрерывная ветвь , принимающая значение , может быть записана в виде функций . Теперь готовы вычислить исходный интеграл

Пример 394.4. Вычислить интеграл , когда контур интегрирования –окружность с началом и обход производится против часовой стрелки. Непрерывная функция выбрана так, что .

Решение. Поскольку , то справедливо представление . Тогда непрерывная ветвь , принимающая значение , может быть записана в виде функций . Теперь готовы вычислить исходный интеграл

Занятие № 6

Тема: Интегральная формула Коши

Нумерация примеров соответствует книге [ 1 ].

Пример 412.1. Вычислить интеграл , когда внутри замкнутого контура[18] интегрирования лежит точка , а точка принадлежит его внешности.

Решение. На лекции доказана интегральная формула Коши

Интегральная формула Коши справедлива для голоморфной на всей плоскости функций . Ее смысл заключается в том, что интеграл пропорционален значению функций. Часто значение функций вычисляется без особых усилий и тогда формула Коши эффективна. Применим формулу Коши для вычисления интеграла . Сначала преобразуем его . На роль претендует выражение . К сожалению, оно не определено (не голоморфно) в точке . По условию примера эта точка лежит во внешности контура . Это положение спасает ситуацию!!! На самом деле, интегральная формула Коши справедлива для голоморфных в некоторой окрестности функций . Поэтому имеем право использования интегральной формулы Коши. В результате . Вместо вычисления интеграла (как это делалось в курсе математического анализа) мы вычислили значение простенькой функций!

Пример 412.2. Вычислить интеграл , когда внутри замкнутого контура интегрирования лежит точка , а точка принадлежит его внешности.

Решение. Применим формулу Коши для вычисления интеграла . Сначала преобразуем его . На роль претендует выражение . В результате . Вместо вычисления интеграла (как это делалось в курсе математического анализа) мы вычислили значение простенькой функций!

Пример 412.3. Вычислить интеграл , когда внутри замкнутого контура интегрирования лежат точка , .

Решение. Применим формулу Коши для вычисления интеграла . Сначала преобразуем его . Каждый из интегралов вычислим по интегральной формуле Коши. В результате искомый интеграл равен нулю.

Пример 412.4. Вычислить интеграл , когда внутри замкнутого контура интегрирования лежат точка , .

Решение. Второй способ решения, основанный на предварительном применении теоремы Коши. После решения примера 386 (занятие 5) была выдвинута гипотеза о том, что иногда один контур можно заменять на другой контур при этом значение интеграла сохраняется. Эта гипотеза на лекции доказана как теорема Коши: в области голоморфности подынтегральной функций контур интегрирования можно непрерывно деформировать при этом значение интеграла сохраняется. В данном примере подынтегральная функция голоморфна в области . Поэтому контур интегрирования можно заменить на другой контур

(смотри рисунок). На самом деле, контур есть объединение двух окружностей с центрами в точках , . То есть мы деформировали контур в контур , не затрагивая точек , , где нарушается голоморфность. По теореме Коши имеем . Остается по интегральной формуле Коши найти, полученные интегралы. Эти интегралы уже вычислены в примерах 412.1 и 412.2. Ответ равен нулю. Советую запомнить: сочетание теоремы Коши и интегральной формулы Коши очень эффективно. Сравните решение данного примера с решением примера 412.3 !!! Здесь нет разложения на элементарные дроби, то есть теорема Коши позволяет без дополнительных аналитических преобразований получить результат.

Пример 413. Вычислить интеграл , когда на замкнутом контуре интегрирования нет точек .

Решение. Надо исследовать все возможные варианты: часть точек из множества может лежать внутри , а остальные — во внешности . Поэтому удобнее отдельно найти интегралы

Если внутри контура интегрирования находятся две точки , то искомый интеграл равен соответствующей сумме интегралов

Важный момент: как из контура получить контур , представляющий объединение двух окружностей с центрами в точках ? Надо восстановить соответствующий рисунок.

Если внутри контура интегрирования находятся две точки , то искомый интеграл равен соответствующей сумме интегралов

Если внутри контура интегрирования находятся три точки , то искомый интеграл равен соответствующей сумме интегралов

Важный момент: как из контура получить контур , представляющий объединение трех окружностей с центрами в точках ? Покажите этот момент на чертеже.

Другие варианты исследуются точно также.

Пример 414. Вычислить интеграл , когда внутри замкнутого контура интегрирования лежат точки , а точки принадлежат его внешности.

Решение. После решения примера 413 этот пример надо решать без комментариев.

Пример 413. Вычислить интеграл .

Решение. 1 шаг. Выяснение тех точек, где нарушается голоморфность.

2 шаг. Расположение найденных точек относительно контура интегрирования. В данном случае точка лежит внутри контура , остальные – во внешности контура .

3 шаг. Вычисление интеграла по интегральной формуле Коши.

Поэтому запишем равенство

Пример 416. Вычислить интеграл , когда внутри замкнутого контура интегрирования лежит окружность .

Решение. 1 шаг. Выяснение тех точек, где нарушается голоморфность.

2 шаг. Расположение найденных точек относительно контура интегрирования. В данном случае обе точки лежат внутри контура интегрирования.

3 шаг. Вычисление интеграла по интегральной формуле Коши.

Пример 417. Вычислить интеграл , когда внутри замкнутого контура интегрирования лежит точка .

Решение. На лекции доказана интегральная формула Коши

Интегральная формула Коши справедлива для голоморфной на всей плоскости функций и произвольной точки . Поэтому, дифференцируя приведенную формулу по параметру , получим новую формулу

Вычислим требуемый интеграл

Пример 418.1. Вычислить интеграл , когда внутри замкнутого контура интегрирования лежат точки .

Решение. Сначала найдем интеграл , а затем вычислим

Для записи полного ответа надо сложить, полученные выражения: .

Вопрос для вдумчивых читательниц: в примере 413 сумма всех трех интегралов, соответствующих всем особым точкам знаменателя, равнялась нулю. В примере 412.4 также сумма всех двух интегралов, соответствующих всем особым точкам знаменателя, равнялась нулю. Здесь сумма всех двух интегралов, соответствующих всем особым точкам знаменателя, не равна нулю. Почему? Ответ прост.[19] Попробуйте найти более мудрый ответ.[20]

Занятие № 7

Тема: Вычеты

Нумерация примеров соответствует книге [ 1 ].

Пример 621. Для данной функций найти ее особые точки, выяснить их типы и определить соответствующие вычеты.

Решение. 1 шаг. Нахождение особых точек.

2 шаг. Типы особых точек.

Исследуем особую точку . Это изолированная особая точка. Действительно, круг содержит лишь одну особую точку . Тогда в окрестности этой точки функцию можно разложить в ряд Лорана. Аккуратно запишем этот факт . В главной части разложения Лорана присутствуют члены, их конечное число, поэтому точка представляет полюс третьего порядка. Исследуем особую точку . Это изолированная особая точка. Действительно, круг содержит лишь одну особую точку . Не разлагая в ряд Лорана, выясним тип особой точки . На лекции доказан критерий полюса: необходимым и достаточным условием полюса является равенство бесконечности предела исследуемой функций при приближений ее аргумента к особой точке. Так как , то — полюс первого порядка.

1.  Исследуем особую точку . Это изолированная особая точка. Действительно, круг содержит лишь одну особую точку . Здесь можно действовать также как в предыдущем пункте, но нам надо научиться разлагать в ряды Лорана. Имеется представление , поэтому сначала отдельно каждое выражение разложим в ряды Лорана, а затем получим разложение . Итак, в окрестности верны разложения

Теперь можем записать разложение

Следовательно, судя по главной части ряда Лорана точка — полюс первого порядка

3 шаг. Нахождение вычетов.

2.  Вычет в точке можно находить двумя способами. Первый способ: использование главной части разложения Лорана (если такое разложение имеется). В нашем случае разложение Лорана уже выписано. Нас интересует коэффициент при , который равен . Следовательно . Второй способ вычисления вычета производится по формуле .

3.  Вычет в точке находим по формуле

4.  Вычет в точке найдем из разложения Лорана. . Отсюда

4 шаг. Исследование точки . С помощью замены надо бесконечность перевести в нуль. В результате имеем функцию , зависящую от . Надо исследовать точку . При справедливо разложение , поэтому — устранимая особая точка. Этот же вывод верен для . Вычет равен нулю.

Вывод: Тип и вычет изолированной особой точки можно определять двумя способами. Либо разлагая исследуемую функцию в соответствующей окрестности в ряд Лорана, либо пользуясь соответствующими критериями особых точек и затем (в полюсах) применить готовые формулы вычетов.

Пример 622. Исследовать особые точки функций

Решение. 1 шаг. Нахождение особых точек.

2 шаг. Типы особых точек.

1)  . Как определяется порядок полюса?

2)  .

3 шаг. Нахождение вычетов.

4 шаг. Исследование точки . С помощью замены надо бесконечность перевести в нуль. В результате имеем функцию , зависящую от . Надо исследовать точку . Справедливо равенство , поэтому — устранимая особая точка. Этот же вывод верен для . Вычет равен нулю.

Пример 623. Исследовать особые точки функций

Решение без комментариев.

Пример 624. Исследовать особые точки функций

Решение без комментариев.

Пример 626. Исследовать особые точки функций

Решение.

В существенно особой точке вычет находится из разложения Лорана. Это не полюс!!! Готовых формул для вычетов я не знаю. Надо каким-то образом вычислить коэффициент разложения Лорана при .

Следовательно, вычет данной функций в бесконечности имеет вид . Сравните с ответом из задачника [1].

Замечание. Не все задачники хороши для проведения занятий. Сборник задач В. П.Демидовича – один из удачных сборников. Чего, к сожалению, нельзя сказать о сборнике по ТФКП [1].

Пример 628. Исследовать особые точки функций .

Решение.

Точно также исследуются другие изолированные особые точки . Заметим, что бесконечно удаленная точка является неизолированной особой точкой, так как . Для неизолированных особых точек мы классификаций не знаем.

Пример 630. Исследовать особые точки функций .

Решение.

Бесконечно удаленная точка является неизолированной особой точкой.

Пример 638. Исследовать особые точки функций .

Решение.

Занятие № 8

Тема: Вычисление контурных интегралов

Нумерация примеров соответствует книге [ 1 ].

Пример 657. Вычислить , где контур интегрирования .

Решение. 1 шаг. Нахождение особых точек подынтегральной функций.

2 шаг. Расположение особых точек относительно контура интегрирования.

Уравнение контура интегрирования представляет уравнение окружности с центром в точке и радиуса . Поэтому две особые точки лежат внутри контура, а две другие – во внешности контура.

3 шаг. Вычисление интеграла по теореме о вычетах.

Согласно названной теореме интеграл . Найдем необходимые вычеты.

Поэтому исходный интеграл

Пример 658. Вычислить , где контур интегрирования .

Решение. 1 шаг. Нахождение особых точек подынтегральной функции.

2 шаг. Расположение особых точек относительно контура интегрирования.

Одна из особых точек лежит внутри контура интегрирования, а другая – во внешности контура интегрирования.

3 шаг. Вычисление интеграла по теореме о вычетах.

Найдем вычет подынтегральной функции в особой точке, принадлежащей внутренности контура интегрирования.

Поэтому интеграл

Пример 659. Вычислить , где контур интегрирования .

Решение. 1 шаг. Нахождение особых точек подынтегральной функции.

2 шаг. Расположение особых точек относительно контура интегрирования.

Особые точки, лежащие на единичной окружности принадлежат внутренности контура интегрирования, а одна точка — во внешности контура. Заметим также, что во внешности контура находится еще одна особая точка .

3 шаг. Вычисление интеграла по теореме о вычетах.

Исходный интеграл можно составить как сумму вычетов соответствующих внутренним особым точкам или как сумму вычетов соответствующих внешним особым точкам. В данном примере лучше использовать внешние вычеты.

Пример 660. Вычислить , где контур интегрирования .

Решение. Надеюсь, что опыт решения предыдущих примеров позволяет сокращенную запись решения данного примера. Во внешности контура интегрирования всего лишь одна особая точка .

Узелок на память[21]. В данном примере бесконечно удаленная точка является устранимой особой точкой подынтегральной функции, однако вычет в этой точке не равен нулю. Такой эффект возможен только в бесконечности!!!

Пример 661. Вычислить , где контур интегрирования .

Решение без комментариев.

Пример 662. Вычислить , где контур интегрирования .

Решение.

Пример 663. Вычислить , где контур интегрирования .

Решение.

Пример 664. Вычислить , где контур интегрирования .

Решение.

Пример 665. Вычислить , где контур интегрирования .

Решение.

Пример 667. Вычислить , где— голоморфные на всей комплексной плоскости функций, а внутри контура интегрирования лежат точки .

Решение. Считаем, что других, кроме приведенных в условии примера, особых точек нет.

Пример 671. Вычислить , где контур интегрирования .

Решение. Особые точки . Будем считать, что . Особые точки лежат внутри контура, так как . В данном случае эффективнее считать вычет на бесконечности, чем вычеты во внутренних особых точках. Считая , разложим функцию по степеням .

Отсюда следует, что или ответ равен 1

Пример 671. Вычислить , где контур интегрирования — вертикальная прямая, проходящая через точку .

Решение. Здесь контур интегрирования не является замкнутой кривой, поэтому теорему о вычетах непосредственно для вычисления исходного интеграла применить нельзя.

Сначала вычислим интеграл по вспомогательному замкнутому контуру , который представлен на рисунке. Это можно сделать с помощью вычетов.

С другой стороны, вспомогательный контур представляет объединение четырех отрезков. Поэтому имеем

Отдельно найдем каждый из выписанных интегралов.

Учитывая полученные значения интегралов, запишем равенство

Итак, исходный интеграл

Занятие № 9

Тема: Вычисление определенных интегралов

Нумерация примеров соответствует книге [ 1 ].

Пример 673. Вычислить определенный интеграл при .

Решение. Цель: определенный интеграл по отрезку свести к интегралу вдоль замкнутого контура и затем применить теорему о вычетах.

Сведение к контурному интегралу.

Вычисление контурного интеграла.

Пример 674. Вычислить определенный интеграл при .

Решение. Сведение к контурному интегралу.

Пример 675. Вычислить определенный интеграл при .

Решение. Сведение к контурному интегралу.

Такой интеграл уже вычислен в предыдущем примере. Поэтому ответ равен

Пример 676. Вычислить определенный интеграл при .

Решение.

Занятие № 10

Тема: Вычисление несобственных интегралов

Нумерация примеров соответствует книге [ 1 ].

Пример 682. Вычислить несобственный интеграл .

Решение. Цель: несобственный интеграл свести к контурному интегралу и затем его вычислить с помощью теоремы о вычетах.

Сведение к контурному интегралу.

Замкнутый контур представляет объединение полуокружности и отрезка . Интеграл по замкнутому контуру вычисляется по теореме о вычетах.

Покажем, что . Для этого достаточно оценить сверху модуль соответствующего интеграла. Если , то .

Таким образом, ответ равен

Вывод: Интеграл по полуокружности стремится к нулю, когда его радиус неограниченно возрастает при условии, что степень числителя намного меньше степени знаменателя.

Пример 683. Вычислить несобственный интеграл .

Решение. Согласно предыдущему выводу имеем

Пример 685. Вычислить несобственный интеграл .

Решение. Согласно предыдущему выводу имеем

Пример 687. Вычислить несобственный интеграл .

Решение. Рассмотрим интеграл

С другой стороны, тот же интеграл можем расписать в виде

Последний интеграл согласно предыдущему выводу стремится к нулю при . Таким образом, исходный интеграл

Пример 691.1. Пользуясь леммой Жордана, вычислить интеграл .

Решение. Так как , то . Поэтому стандартным образом (смотри решение примера 682) исходный интеграл сведем к контурному интегралу.

Замкнутый контур представляет объединение полуокружности и отрезка . Интеграл по замкнутому контуру вычисляется по теореме о вычетах.

Покажем, что интеграл . Для этого достаточно оценить сверху модуль соответствующего интеграла. Если , то .

Иногда полученный факт стремления к нулю подобных интегралов называют леммой Жордана. Обратите внимание, что здесь интеграл стремится к нулю за счет экспоненциального множителя. Иначе дело обстояло в примере 682. В результате имеем

Пример 697. Вычислить интеграл .

Решение. Данный пример отличается от предыдущего тем, что нули знаменателя подынтегральной функции принадлежат прямой интегрирования. Действительно, выражение равно нулю при , которые лежат на оси интегрирования . Поэтому несколько модифицируем метод сведения исходного интеграла к контурному интегралу, который был проиллюстрирован в примере 682. Введем обозначения

— верхняя полуокружность с центром в нуле и радиуса , — верхняя полуокружность с центром в 2 и радиуса , — верхняя полуокружность с центром в 3 и радиуса

Ориентации полуокружностей против часовой стрелки. Сведение к контурному интегралу

Здесь — замкнутый контур представляет объединение трех отрезков вещественной оси и трех полуокружностей

Остается вычислить четыре интеграла

  по ИТК,

 

 

  по лемме Жордана (смотри решение примера 691.1)

Отсюда можно записать ответ

Занятие № 11

Тема: Дробно-линейные отображения

Нумерация примеров соответствует книге [ 1 ].

Пример 193. Найти целую линейную функцию, отображающую треугольник с вершинами в точках на подобный ему треугольник с вершинами .

Решение. Цель: научиться мыслить без формул на основе геометрических образов. Надо освоить язык рисунков.

Указанным геометрическим действиям соответствуют следующие формулы:

  — поворот на ,

  — горизонтальный сдвиг,

  — растяжение с поворотом.

Окончательный ответ примет вид .

Пример 194. Найти целое линейное преобразование с неподвижной точкой , переводящее точку в точку .

Решение. Заметим, что отображение имеет неподвижную точку . Воспользуемся этим фактом.

Указанным геометрическим действиям соответствуют следующие формулы:

  ,

  ,

  .

Окончательный ответ примет вид .

Пример 200.1. Для функции найти образ следующей линии .

Решение. Дробно-линейное отображение переводит окружность в окружность. Найдем эту окружность. Точка лежит на окружности , ее образом при отображении будет бесконечно удаленная точка. Итак, образом является окружность, содержащая бесконечно удаленную точку, то есть образ – прямая. Остается указать эту прямую. Чтобы построить прямую, достаточно знать одну ее точку и ее направление. Точка лежит на окружности , ее образом при отображении будет точка . Следовательно, искомая прямая проходит через точку . Остается указать направление прямой. Окружность в точке перпендикулярна прямой . Поскольку при конформных отображениях углы сохраняются, то искомая прямая должна оставаться перпендикулярной образу прямой при отображении . Если , то — также остается вещественной, то есть представляет вещественную ось. Итак, искомая прямая проходит через точку и перпендикулярна вещественной оси. То есть образ окружности представляет прямую параллельную мнимой оси.

Пример 203. Во что преобразуется квадрант при отображении .

Решение. Воспользуемся принципом соответствия границ, то есть образ границы – граница образа. Найдем образ границы квадранта.

  Сначала рассмотрим границу . Если , то расстояния от точки до точек одинаковы. Следовательно, , что означает – дугу единичной окружности на . Если к тому же , то . Если , то . Таким образом, граница при отображении переходит в единичную полуокружность с началом в точке и конечной точкой .

  Теперь рассмотрим другую границу . Прямая при отображении переходит в прямую, лежащую на и проходящую через точку . Это следует из того, что при имеем , а при имеем . Если , то . Таким образом, прямая при отображении переходит в прямую, лежащую на и проходящую через точки . Отсюда заключаем, что полупрямая переходит в отрезок . Окончательный ответ: квадрант переходит в нижнюю единичную полуокружность, так как в точке должен сохранятся ориентированный угол .

Пример 209. Найти дробно-линейное отображение, переводящее точки соответственно в точки .

Решение.

  Построим дробно-линейное отображение, переводящее заданные точки в точки . Это легко сделать. Действительно, отображение переводит точки в точки . Остается добиться условия . Необходимое отображение ищем в виде . Из условия следует, что . Отсюда следует .

  Построим дробно-линейное отображение, переводящее заданные точки в точки . Это легко сделать. Действительно, отображение переводит точки в точки . Остается добиться условия . Из условия следует, что . Отсюда следует .

  Теперь остается приравнять найденные отображения . Надо определить . Итак, имеем .

Пример 213.1. Найти общий вид дробно-линейного преобразования, переводящего верхнюю полуплоскость на себя.

Решение. Согласно принципу соответствия границ – граница полуплоскости переходит в границу полуплоскости. То есть вещественная ось преобразуется в вещественную ось . Обозначим прообразы соответственно через , причем введенные числа вещественны. Изучим возможные варианты.

  Пусть , то есть . Следовательно, преобразование имеет вид . Если , то , где должно быть вещественным. Так как из должно следовать , поэтому .

  Пусть . Если , то , где — вещественное число. Ориентация тройки должна сохраняться, то есть должно быть отрицательным числом. Итак, получилась система неравенств

Так как , то или .

  Пусть . В этом случае приходим к системе

Отсюда следует, что

Окончательный ответ: .

Пример 216.1. Найти точку симметричную с точкой относительно единичной окружности.

Решение. Вспомним, что симметрия точек при дробно-линейных отображениях сохраняется. Симметрия относительно прямой наглядна. Поэтому преобразуем окружность с помощью дробно-линейного отображения в прямую. Затем найдем точку, симметричную образу заданной точки. Остается сделать обратное преобразование.

  Дробно-линейное отображение преобразует единичную окружность в прямую, проходящую через точки .Заметим, что симметричны относительно единичной окружности и их образы соответственно имеют вид . Следовательно, прямая проходит через точку и перпендикулярна отрезку . Иначе говоря, прямая проходит через точку и вертикальна.

  При этом точка переходит в точку .

  Точке симметричной относительно вертикальной прямой, проходящей через , будет точка

  Из уравнения найдем .

Занятие № 12

Тема: Конформные отображения

Нумерация примеров соответствует книге [ 1 ].

Пример 272.1. При помощи функции и ей обратной найти конформное отображение внутренности правой ветви равнобочной гиперболы на верхнюю полуплоскость.

Решение. Если , то . Поэтому отображение переводит требуемую область на область . Остается эту область повернуть против часовой стрелки на . Ответ .

Пример 272.2. При помощи функции и ей обратной найти конформное отображение внешности параболы на верхнюю полуплоскость.

Решение. Отметим, что можно пользоваться как прямым, так и обратным отображением. Если и , то

Если и , то

Воспользуемся обратным преобразованием . Если , то . Возьмем в качестве , тогда . Но нам необходимо получить , поэтому .

Пример 278.1. Отобразить угол на верхнюю полуплоскость.

Решение. Применим принцип соответствия границ.

  Образ границы должен совпадать с лучом .

  Образ границы должен совпадать с лучом .

Достаточно положить .

Пример 279.1. Найти функцию , отображающую полукруг на верхнюю полуплоскость при условиях .

Решение. В предыдущем примере угол конформно отображен на полуплоскость. Здесь полуокружность надо перевести на угол, а затем угол отобразить на полуплоскость.

  Найдем дробно-линейное отображение, которое полуокружность переводит в угол. Положим , тогда точки переходят соответственно . Так как отрезок и верхняя единичная полуокружность переходят соответственно в лучи . То есть переходит в угол раствора .

  Остается отобразить угол на полуплоскость. Положим .

  Надо проверить, что все условия выполняются.

Пример 281. Найти функцию , отображающую область на верхнюю полуокружность.

Решение. Достаточно вспомнить ответ предыдущего примера. Легко понять, что требуемую область отображение переводит в нижнюю полуплоскость. Остается нижнюю полуплоскость отобразить в верхнюю, то есть . Пример имеет множество ответов.

Пример 282.1. Отобразить на верхнюю полуплоскость сектор .

Решение. Сначала сектор отобразим в полуокружность, а затем полуокружность – в полуплоскость.

  , смотри пример 278.1,

  , смотри пример 279.1.

Пример 283.1. Отобразить на верхнюю полуплоскость круговую луночку .

Решение. Сначала луночку отобразим в угол, а затем угол – в полуплоскость. При этом надо знать угловые точки и углы луночки. Нетрудно понять, что угловые точки имеют вид и угол равен .

  , причем выберем из условия ,

 

Окончательный ответ: .

Пример 285. Отобразить плоскость с разрезом по отрезку на верхнюю полуплоскость.

Решение. Сначала растянем разрез так, чтобы получилась плоскость с разрезом по лучу. Затем ее отобразим на полуплоскость.

 

 

Пример 290. Отобразить плоскость с разрезом по дуге окружности, соединяющей точки и проходящей через точку , на верхнюю полуплоскость.

Решение. Сначала растянем разрез так, чтобы получилась плоскость с разрезом по лучу. Затем ее отобразим на полуплоскость.

  , причем выберем из условия

 

Занятие № 13

Тема: Распределение нулей

Нумерация примеров соответствует книге [ 1 ].

Пример 788. Пользуясь теоремой Руше, найти количество лежащих внутри круга корней уравнения .

Решение.

так как при модуль подчиненной части не превосходит четырех, а модуль главной части не меньше шести. Иначе говоря, на единичной окружности модуль подчиненной части меньше модуля главной части. Главная часть в единичном круге имеет ровно один нуль. Следовательно, по теореме Руше исходное уравнение в единичном круге также имеет один корень. Разложение на главную и подчиненную части неоднозначно. К примеру,

Здесь при справедливы оценки

В данном случае главная часть в единичном круге имеет один нуль. Следовательно, по теореме Руше исходное уравнение в единичном круге также имеет один корень.

Пример 790. Пользуясь теоремой Руше, найти количество лежащих внутри круга корней уравнения .

Решение.

так как при модуль подчиненной части не превосходит четырех, а модуль главной части не меньше пяти. Иначе говоря, на единичной окружности модуль подчиненной части меньше модуля главной части. Главная часть в единичном круге имеет ровно четыре нуля. Следовательно, по теореме Руше исходное уравнение в единичном круге также имеет четыре корня.

Пример 792. Сколько корней уравнения находится в кольце .

Решение. Сначала выясним сколько корней находится в единичном круге, а затем сколько корней в круге .

 

так как при модуль подчиненной части не превосходит двух, а модуль главной части не меньше пяти. Иначе говоря, на единичной окружности модуль подчиненной части меньше модуля главной части. Главная часть в единичном круге имеет ровно один нуль. Следовательно, по теореме Руше исходное уравнение в единичном круге также имеет один корень.

 

так как при модуль подчиненной части не превосходит одиннадцати, а модуль главной части не меньше тридцати двух. Иначе говоря, на окружности модуль подчиненной части меньше модуля главной части. Главная часть в круге имеет ровно четыре нуля. Следовательно, по теореме Руше исходное уравнение в круге также имеет четыре корня.

  Следовательно, в кольце исходное уравнение имеет три корня.

Пример 804.1. Найти количество нулей многочлена в правой полуплоскости.

Решение. Применим принцип аргумента. Для этого надо вычислить приращение аргумента многочлена вдоль мнимой оси и вдоль правой полуокружности достаточно большого радиуса.

  Приращение аргумента вдоль мнимой оси.

Нам потребуются нули мнимой и реальной частей выписанного в скобках выражения.

В таблице показано изменение знаков реальной и мнимой частей.

+

+

+

+

четверть

4

3

4

1

+

+

+

+

+

+

четверть

4

1

2

1

Когда изменяется от до , то вектор из первой четверти перемещается в четвертую. Следовательно, может принимать значения между нулем и .

  Приращение аргумента вдоль полуокружности. Пусть .

Заметим, что

Таким образом, приращение

  Приращение вдоль замкнутого контура, состоящего из полуокружности и вертикального отрезка мнимой оси, может принимать значения от до . Но приращение аргумента вдоль замкнутого контура, деленное на , обязательно должно быть целым числом. В данном случае это число равно 2. Итак, согласно принципу аргумента число нулей исходного многочлена в правой полуплоскости равно 2.

Пример 804.2. Найти количество нулей многочлена в первой четверти.

Решение. Надо вычислить приращение аргумента многочлена вдоль границы первой четверти. Так как все нули многочлена находятся внутри круга , то достаточно найти количество нулей попадающих в сектор .

  Приращение аргумента вдоль мнимой полуоси. Из таблицы, которая приведена в предыдущем примере, имеем: приращение аргумента не более .

  Приращение аргумента вдоль дуги, представляющую часть границы сектора, равно . Смотри решение предыдущего примера.

  Остается найти приращение аргумента вдоль вещественной полуоси. Ясно, что данный многочлен при положительных значениях принимает только положительные значения. Следовательно, приращения не будет, то есть оно равно нулю.

  Ответ: в первой четверти данный многочлен имеет один нуль.

Пример 796. Сколько корней уравнения находится в круге .

Решение.

так как при справедливы оценки

Главная часть внутри единичного круга имеет ровно нулей. Поэтому исходное уравнение внутри единичного круга также имеет ровно корней.

Занятие № 14

Тема: Применение методов ТФКП

Цель занятия: показать некоторые приложения методов комплексного анализа в различных разделах математики.

Пример: устойчивые многочлены. Многочлен называется устойчивым, если все его нули лежат в левой полуплоскости. Будет ли заданный многочлен устойчивым?

Решение подобных вопросов проводится также, как это делалось на занятий 13.

Пример: собственные значения матриц. Где лежат собственные значения заданной матрицы?

Решение проводим по следующей схеме.

  Строим характеристический многочлен , где — заданная матрица, — единичная матрица.

  Методы занятия 13 позволяют исследовать нули многочлена .

  Нули характеристического многочлена являются собственными значениями матрицы.

Пример: собственные значения краевой задачи для дифференциальных уравнений. Где лежат собственные значения заданного дифференциального оператора? В качестве конкретного примера рассмотрите краевую задачу

Решение. Общее решение уравнения имеет вид , где — произвольные числа. Из условия следует, что . Остается удовлетворить условие . В результате имеем . Таким образом, нули целой функции представляют собственные значения исходной краевой задачи. Явных формул для нулей нет. Нули целых функций – изолированы, они не могут иметь конечных предельных точек. Эти утверждения доказаны на лекциях. Попытаемся доказать, что их счетное число. Уравнение можно переписать в виде . Вещественные решения последнего уравнения можно найти графическим способом.

Пересечение двух графиков представляют собственные значения краевой задачи. В каждой вертикальной полосе по одной точке пересечения. Таким образом, точек пересечения – счетное число. Утверждение доказано.

Вывод: собственные значения матриц — это нули многочленов, собственные значения краевых задач для дифференциальных уравнений на отрезке – это нули целых функций.

Пример: асимптотическое поведение нулей квазиполиномов. Функция вида называется квазиполиномом. Найти асимптотическое поведение нулей заданного квазиполинома.

Подобные проблемы возникают при инженерных расчетах. Если чисто мнимые числа, то задача нахождения нулей квазиполинома эквивалентна решению тригонометрических уравнений. К примеру, тригонометрическое уравнение эквивалентно нахождению нулей квазиполинома вида . Если обозначить через , то получим многочлен , который может иметь 70 нулей. Если его нули, то каждый нуль многочлена порождает свою серию нулей квазиполинома.

.

Аналогичные утверждения можно получить для произвольных квазиполиномов. Не вдаваясь в детали, приведу один полезный алгоритм нахождения нулей квазиполинома. Этот алгоритм пригоден только для квазимногочленов специального вида.

Пусть набор таков, что является множеством всех вершин некоторого выпуклого многоугольника. Тогда достаточно большие по модулю нули квазиполинома имеют вид

К примеру, нули квазиполинома находим по формулам

Несмотря на то, что мы не знаем чисел , приведенные формулы довольно эффективны. Так как достаточно велико, то большинство значащих цифр нулей определяется первыми слагаемыми приведенных формул.

Пример: теорема Миттаг-Леффлера. Любая рациональная функция разлагается на сумму некоторого многочлена и элементарных дробей, представляющих главные части в конечных особых точках. Доказать.

Решение. Эта теорема применялась при вычислении неопределенных интегралов от рациональных функций. Здесь приводим доказательство этого утверждения, которое основано на методах ТФКП. Отметим, что приведенное доказательство распространяется на мероморфные функций, то есть по главным частям в особых точках можно восстановить мероморфную функцию. Через обозначим полюсы рациональной функции , а через главную часть ее лорановского разложения в полюсе . Тогда разность представляет целую функцию, которая на бесконечности имеет степенной рост. Из теоремы Лиувилля следует, что разность — многочлен. Аналогичные рассуждения пригодны для меромофных функций. Если , то . Разность представляет целую функцию. Можно доказать, что на бесконечности эта функция стремиться к нулю. Тогда по теореме Лиувилля эта разность равна нулю. Таким образом, имеем равенство..

ЧАСТЬ 3

ТЕСТЫ

Тесты на уровне определений.

1. На каком из следующих множеств

A) , B) , C) , D) , E)

уравнение не имеет решений?

2. На каком из следующих множеств

A) , B) , C) , D) , E)

уравнение не имеет решений?

3. На каком из следующих множеств

A) , B) , C) , D) , E)

уравнение не имеет решений?

4. На каком из следующих множеств

A) , B) , C) , D) , E)

уравнение не имеет решений?

5. Пусть — различные квадраты с центром в нуле и сторонами параллельными координатным осям, причем . Если лежат на комплексной плоскости , то какое из утверждений

A) , B) , C) ,

D) , E)

верно для чисел .

6. Пусть — различные квадраты с центром в нуле и сторонами параллельными координатным осям, причем . Если лежат на комплексной плоскости , то какое из утверждений

A) , B) , C) ,

D) , E)

верно для чисел .

7. Пусть — различные квадраты с центром в нуле и сторонами параллельными координатным осям, причем . Если лежат на комплексной плоскости , то какое из утверждений

A) , B) , C) , D) , E)

верно для чисел .

8. Пусть — две вертикальные полосы, симметричные относительно мнимой оси, причем . Если лежат на комплексной плоскости , то какое из утверждений

A) , B) , C) , D) , E)

верно для чисел .

9. Пусть — две горизонтальные полосы, симметричные относительно вещественной оси, причем . Если лежат на комплексной плоскости , то какое из утверждений

A) , B) , C) , D) , E)

верно для чисел .

10. Пусть — различные круги с центром в начале координат, причем . Если лежат на комплексной плоскости , то какое из утверждений

A) , B) , C) , D) , E)

верно для чисел .

11. Пусть — горизонтальная полоса ширины , симметричная относительно вещественной оси. Если комплексное число лежит в , то верно

A) , B) , C) , D) , E)

12. Пусть — вертикальная полоса ширины , симметричная относительно мнимой оси. Если комплексное число лежит в , то верно

A) , B) , C) , D) , E)

13. Пусть — круг с центром в начале координат и радиуса 18. Если комплексное число лежит в , то верно

A) , B) , C) , D) , E)

14. Пусть — квадрат с центром в нуле и сторонами параллельными координатным осям, причем ширина квадрата равна 20. Если комплексное число лежит в , то верно

A) , B) , C) ,

D) , E)

15. Пусть — квадрат с центром в нуле и сторонами параллельными координатным осям, причем ширина квадрата равна 20. Если комплексное число лежит в , то верно

A) , B) , C) ,

D) , E)

16. Пусть — окружность с центром в начале координат и радиуса 18. Если комплексное число лежит на , то верно

A) , B) , C) , D) , E)

17. Сколько разных корней существует, лежащих в первой четверти?

A) , B) , C) , D) , E)

18. Сколько разных корней существует, лежащих в верхней полуплоскости?

A) , B) , C) , D) , E)

19. Сколько разных корней существует, лежащих в правой полуплоскости?

A) , B) , C) , D) , E)

20. Сколько разных корней существует, лежащих в левой полуплоскости?

A) , B) , C) , D) , E)

21. Сколько разных корней существует, лежащих в единичном круге?

A) , B) , C) , D) , E)

22. Сколько разных корней существует, лежащих в единичном круге?

A) , B) , C) , D) , E)

23. Какое из утверждений

A) , B) , C) ,

D) , E)

верно для чисел ?

24. Какое из утверждений

A) , B) , C) ,

D) , E)

верно для чисел ?

25. Какое из утверждений

A) , B) , C) ,

D) , E)

верно для чисел ?

26. Какое из утверждений

A) , B) , C) ,

D) , E)

верно для чисел ?

27. Какое из утверждений

A) лежат в третьей четверти,

B)хотя бы одно из них принадлежит первой четверти,

C) хотя бы одно из них принадлежит второй четверти,

D) хотя бы одно из них принадлежит четвертой четверти,

E) принадлежат кругу с центром в нуле и радиуса 17

для набора чисел не выполняется?

28. Какое из утверждений

A) принадлежат кругу с центром в нуле и радиуса ,

B)хотя бы одно из них принадлежит первой четверти,

C) хотя бы одно из них принадлежит второй четверти,

D) хотя бы одно из них принадлежит четвертой четверти,

E) принадлежат кругу с центром в нуле и радиуса 17

для набора чисел не выполняется?

29. На комплексной плоскости выбрана — вертикальная полоса ширины , симметричная относительно мнимой оси. Тогда означает

A) вертикальную полосу ширины , симметричную относительно мнимой оси,

B) вертикальную полосу ширины , лежащую целиком в правой полуплоскости,

C) горизонтальную полосу ширины , симметричную относительно вещественной оси,

D) горизонтальную полосу ширины , лежащую целиком в верхней полуплоскости,

E) горизонтальную полосу ширины , лежащую целиком в нижней полуплоскости.

30. На комплексной плоскости выбрана — вертикальная полоса ширины , симметричная относительно мнимой оси. Тогда означает

A) вертикальную полосу ширины , симметричную относительно мнимой оси,

B) вертикальную полосу ширины , лежащую целиком в правой полуплоскости,

C) горизонтальную полосу ширины , симметричную относительно вещественной оси,

D) горизонтальную полосу ширины , лежащую целиком в верхней полуплоскости,

E) горизонтальную полосу ширины , лежащую целиком в нижней полуплоскости.

31. На комплексной плоскости выбрана — вертикальная полоса ширины , симметричная относительно мнимой оси. Тогда означает

A) вертикальную полосу ширины , симметричную относительно мнимой оси,

B) вертикальную полосу ширины , лежащую целиком в правой полуплоскости,

C) горизонтальную полосу ширины , симметричную относительно вещественной оси,

D) горизонтальную полосу ширины , лежащую целиком в верхней полуплоскости,

E) горизонтальную полосу ширины , лежащую целиком в нижней полуплоскости.

32. На комплексной плоскости выбрана — вертикальная полоса ширины , симметричная относительно мнимой оси. Тогда означает

A) вертикальную полосу ширины , симметричную относительно мнимой оси,

B) вертикальную полосу ширины , лежащую целиком в правой полуплоскости,

C) горизонтальную полосу ширины , симметричную относительно вещественной оси,

D) горизонтальную полосу ширины , лежащую целиком в верхней полуплоскости,

E) горизонтальную полосу ширины , лежащую целиком в нижней полуплоскости.

33. На комплексной плоскости выбрана — вертикальная полоса ширины , симметричная относительно мнимой оси. Тогда означает

A) вертикальную полосу ширины , симметричную относительно мнимой оси,

B) вертикальную полосу ширины , лежащую целиком в правой полуплоскости,

C) горизонтальную полосу ширины , симметричную относительно вещественной оси,

D) горизонтальную полосу ширины , лежащую целиком в верхней полуплоскости,

E) горизонтальную полосу ширины , лежащую целиком в нижней полуплоскости.

34. На комплексной плоскости выбрана — вертикальная полоса ширины , симметричная относительно мнимой оси. Тогда означает

A) вертикальную полосу ширины , симметричную относительно мнимой оси,

B) вертикальную полосу ширины , лежащую целиком в правой полуплоскости,

C) горизонтальную полосу ширины , симметричную относительно вещественной оси,

D) горизонтальную полосу ширины , лежащую целиком в верхней полуплоскости,

E) горизонтальную полосу ширины , лежащую целиком в нижней полуплоскости.

35. Пусть — действительное число и . Тогда из формулы Эйлера следует, что

A) ,

B) ,

C) ,

D) имеет наименьший период равный ,

E) принимает только действительные значения.

36. Пусть — действительное число и . Тогда из формулы Эйлера следует, что

A) ,

B) ,

C) ,

D) имеет наименьший период равный ,

E) принимает только действительные значения.

37. Пусть — действительное число и . Тогда из формулы Эйлера следует, что

A) ,

B) ,

C) ,

D) имеет наименьший период равный ,

E) принимает только действительные значения.

38. Пусть — действительное число и . Тогда из формулы Эйлера следует, что

A) ,

B) ,

C) ,

D) имеет наименьший период равный ,

E) принимает только действительные значения.

39. Пусть — действительное число и . Тогда из формулы Эйлера следует, что

A) ,

B) ,

C) ,

D) имеет наименьший период равный ,

E) принимает только действительные значения.

40. Модуль числа равен

A) ,

B) ,

C) ,

D) ,

E) .

41. Модуль числа равен

A) ,

B) ,

C) ,

D) ,

E) .

42. Модуль числа равен

A) ,

B) ,

C) ,

D) ,

E) .

43. Модуль числа равен

A) ,

B) ,

C) ,

D) ,

E) .

44. Аргумент числа равен

A) ,

B) ,

C) ,

D) ,

E) .

45. Аргумент числа равен

A) ,

B) ,

C) ,

D) ,

E) .

46. Аргумент числа равен

A) ,

B) ,

C) ,

D) ,

E) .

47. Аргумент числа равен

A) ,

B) ,

C) ,

D) ,

E) .

48. Аргумент числа, сопряженного к числу , равен

A) ,

B) ,

C) ,

D) ,

E) .

49. Формула Муавра имеет вид

A) ,

B) ,

C) ,

D) ,

E) .

50. Формула Эйлера имеет вид

A) ,

B) ,

C) ,

D) ,

E) .

51. Формула Коши-Адамара имеет вид

A) ,

B) ,

C) ,

D) ,

E) .

52. Ряд Тейлора имеет вид

A) ,

B) ,

C) ,

D) ,

E) .

53. Ряд Лорана имеет вид

A) ,

B) ,

C) ,

D) ,

E) .

54. Ряд Пюизе имеет вид

A) ,

B) ,

C) ,

D) ,

E) .

55. Теорема. Интеграл от голоморфной в односвязной области функции вдоль любого замкнутого контура равен нулю. Сформулирована

A) теорема Римана,

B) теорема Монтеля,

C) интегральная теорема Коши,

D) теорема Мореры,

E) теорема о монодромии.

56. Теорема. Если функция непрерывна в области и интеграл от нее по границе любого треугольника равен нулю, то . Сформулирована

A) теорема Римана,

B) теорема Монтеля,

C) интегральная теорема Коши,

D) теорема Мореры,

E) теорема о монодромии.

57. Теорема. Если некоторый элемент аналитически продолжаем вдоль любого пути , принадлежащего односвязной области , то определяемая продолжениями элемента в области аналитическая функция однозначна в этой области. Сформулирована

A) теорема Римана,

B) теорема Монтеля,

C) интегральная теорема Коши,

D) теорема Мореры,

E) теорема о монодромии.

58. Теорема. Из любой равномерно ограниченного внутри семейства голоморфных функций в области можно выделить равномерно сходящуюся на любом компакте из последовательность. Сформулирована

A) теорема Римана,

B) теорема Монтеля,

C) интегральная теорема Коши,

D) теорема Мореры,

E) теорема о монодромии.

59. Теорема. Любая односвязная область, граница которой содержит более одной точки, изоморфна единичному кругу. Сформулирована

A) теорема Римана,

B) теорема Монтеля,

C) интегральная теорема Коши,

D) теорема Мореры,

E) теорема о монодромии.

60. Теорема. Пусть последовательность , голоморфных в области , равномерно на любом компакте из сходится к непостоянной функции . Тогда, если , то в любом круге все функции (при некотором ) обращаются в нуль. Сформулирована

A) теорема Лиувилля,

B) теорема Гурвица,

C) интегральная теорема Коши,

D) теорема Руше,

E) теорема Лорана.

61. Теорема. Если удается разложить на сумму , так что

    — голоморфная функция в , на границе области справедлива оценка или ,

то . Таким образом, вместо того чтобы решать задачу 2 для уравнения достаточно решить задачу 2 для уравнения . Сформулирована

A) теорема Лиувилля,

B) теорема Гурвица,

C) интегральная теорема Коши,

D) теорема Руше,

E) теорема Лорана.

62. Теорема. Сумма ряда Лорана голоморфна в некотором кольце. Сформулирована

A) теорема Лиувилля,

B) теорема Гурвица,

C) интегральная теорема Коши,

D) теорема Руше,

E) теорема Лорана.

63. Теорема. Целая функция, модуль которой растет не быстрее некоторой степени модуля своего аргумента, представляет многочлен. Сформулирована

A) теорема Лиувилля,

B) теорема Гурвица,

C) интегральная теорема Коши,

D) теорема Руше,

E) теорема Лорана.

64. Теорема. Пусть непересекающиеся области имеют общий прямолинейный участок границы , а функций соответственно голоморфны в и непрерывны на множествах . Тогда, если , то функция голморфна в области .

Сформулирован

A) принцип симметрии,

B) принцип аргумента,

C) принцип сохранения области,

D) принцип максимума модуля,

E) теорема Лорана.

65. Теорема. Модуль голоморфной в области функции достигает максимума во внутренней точке тогда и только тогда, когда функция – постоянна. Сформулирован

A) принцип симметрии,

B) принцип аргумента,

C) принцип сохранения области,

D) принцип максимума модуля,

E) теорема Лорана.

66. Теорема. Образ непостоянной голоморфной функции определенной на области представляет область. Сформулирован

A) принцип симметрии,

B) принцип аргумента,

C) принцип сохранения области,

D) принцип максимума модуля,

E) теорема Лорана.

67. Теорема. Интеграл равен деленному на приращению аргумента функции при однократном обходе ориентированной границы подобласти .

. Сформулирован

A) принцип симметрии,

B) принцип аргумента,

C) принцип сохранения области,

D) принцип максимума модуля,

E) теорема Лорана.

68. Теорема. Пусть — изолированная особая точка функции . Следующие предложения эквивалентны:

— устранимая особая точка, существует конечный предел Сформулирован

A) критерии устранимой особой точки,

B) критерии полюса,

C) критерии существенно особой точки,

D) принцип максимума модуля,

E) критерии Коши.

69. Теорема. Пусть — изолированная особая точка функции . Следующие предложения эквивалентны:

— полюс, существует бесконечный предел . Сформулирован

A) критерии устранимой особой точки,

B) критерии полюса,

C) критерии существенно особой точки,

D) принцип максимума модуля,

E) критерии Коши.

70. Теорема. Пусть — изолированная особая точка функции . Следующие предложения эквивалентны:

— существенно особая точка, не существует предела . Сформулирован

A) критерии устранимой особой точки,

B) критерии полюса,

C) критерии существенно особой точки,

D) принцип максимума модуля,

E) критерии Коши.

71. Интегральная формула Коши имеет вид

A) ,

B) ,

C) ,

D) ,

E) .

72. Общий вид дробно-линейной функций имеет вид

A) ,

B) ,

C) ,

D) ,

E) .

Тесты для подготовки к контрольным работам.

1. Выполните следующую операцию над комплексными числами: .

Решение. Использование алгебраической формы записи комплексных чисел. На лекции утверждалось, что каждое комплексное число имеет сопряженное число . Тогда справедливы равенства

A) ,

B) , ,

C) , ,

D) , ,

E) , .

2. .Найдите модуль и аргумент комплексного числа : .

Решение. Так как

то . Поэтому .

A) , , ,

B) , , ,

C) , , ,

D) , , ,

E) , , .

3. Вычислить сумму тригонометрических функций: .

Решение. Поскольку , то

A) , , ,

B) , , ,

C) , , ,

D) , , ,

E) , , .

4. Вычислить все значения логарифма: .

Решение. Обозначим искомый корень через . Тогда

A) , ,

B) , ,

C) , ,

D) , ,

E) , .

5. При каких значениях параметров функция будет голоморфной?

Решение. На лекции утверждалось, что частные производные реальной и мнимой частей голоморфных функций обязаны удовлетворять условиям Коши-Римана. Поэтому сначала выделим реальную и мнимую части данной функций, а затем вычислим их частные производные и выберем параметры так, чтобы выполнились условия Коши-Римана. 1 шаг. Нахождение .

2 шаг. Вычисление производных.

3 шаг. Выбор параметров. Запишем условия Коши-Римана

Поэтому будем выбирать параметры согласно системе

Отсюда имеем . Таким образом, исключая параметр из определения функции , получаем

A) , , ,

B) , , ,

C) , , ,

D) , , ,

E) , , .

6. Зная реальную часть голоморфной функции, надо восстановить голоморфную функцию.

Решение. Так как , то достаточно найти ее мнимую часть. Из условии Коши-Римана следует, что можно считать известными частные производные мнимой части. Таким образом, надо по частным производным найти неизвестную функцию.

1 шаг. Нахождение частных производных .

2 шаг. Вычисление мнимой части по формуле

, ее многие знают из курса математического анализа. Точку в данном случае можно выбирать по своему усмотрению. Пусть . Еще один важный момент. Форма контура интегрирования также зависит от Вашего настроения. Контур показан жирной ломанной линией на рисунке. Ломанная линия состоит из двух отрезков, поэтому

3 шаг. Построение искомой функции.

A) , , ,

B) , , ,

C) , , ,

D) , , ,

E) , , .

7. Выяснить геометрический смысл модуля и аргумента производной следующей функции при .

Решение. Найдем производную в точке :

Запишем приращение функции в окрестности точки

Обозначим через значение функции . Тогда последнее равенство примет вид

Требуемый геометрический смысл вытекает из соотношения .

·  Если пробегает по окружности достаточно малого радиуса, то из предыдущей формулы вытекает, что пробегает некоторую кривую на плоскости «близкую» к окружности . То есть происходит … – это за счет модуля производной .

·  Применяя к обеим частям операцию , получаем равенство . Следовательно, приращение почти совпадает с приращением . Это связано с тем, что . То есть происходит поворот на ноль радиан.

A) , растяжение в три раза,

B) , сжатие в три раза,

C) , растяжение в три раза,

D) , растяжение в два раза,

E) , растяжение в три раза.

8. Вычислить интеграл , когда внутри замкнутого контура интегрирования лежит точка , а точка принадлежит его внешности.

Решение. Применим формулу Коши для вычисления интеграла . Сначала преобразуем его . На роль претендует выражение . В результате .

A) , ,

B) , ,

C) , ,

D) , ,

E) , .

9. Вычислить интеграл , когда на замкнутом контуре интегрирования нет точек .

Решение. Надо исследовать все возможные варианты: часть точек из множества может лежать внутри , а остальные — во внешности . Поэтому удобнее отдельно найти интегралы

Если внутри контура интегрирования находятся две точки , то искомый интеграл равен соответствующей сумме интегралов

Если внутри контура интегрирования находятся две точки , то искомый интеграл равен соответствующей сумме интегралов

Если внутри контура интегрирования находятся три точки , то искомый интеграл равен соответствующей сумме интегралов

A) , ,

B) , ,

C) , ,

D) , ,

E) , .

10. Вычислить интеграл .

Решение. 1 шаг. Выяснение тех точек, где нарушается голоморфность.

2 шаг. Расположение найденных точек относительно контура интегрирования. В данном случае точка лежит внутри контура , остальные – во внешности контура .

3 шаг. Вычисление интеграла по интегральной формуле Коши.

Поэтому запишем равенство

A) ,

B) ,

C) ,

D) ,

E) .

11. Для данной функций найти ее особые точки, выяснить их типы и определить соответствующие вычеты.

Решение. 1 шаг. Нахождение особых точек.

2 шаг. Типы особых точек.

Исследуем особую точку . Это изолированная особая точка. Действительно, круг содержит лишь одну особую точку . Тогда в окрестности этой точки функцию можно разложить в ряд Лорана. Аккуратно запишем этот факт . В главной части разложения Лорана присутствуют члены, их конечное число, поэтому точка представляет полюс третьего порядка. Исследуем особую точку . Это изолированная особая точка. Действительно, круг содержит лишь одну особую точку . Не разлагая в ряд Лорана, выясним тип особой точки . На лекции доказан критерий полюса: необходимым и достаточным условием полюса является … при приближений ее аргумента к особой точке. Так как , то — полюс первого порядка.

5.  Исследуем особую точку . Это изолированная особая точка. Действительно, круг содержит лишь одну особую точку . Здесь можно действовать также как в предыдущем пункте, но нам надо научиться разлагать в ряды Лорана. Имеется представление , поэтому сначала отдельно каждое выражение разложим в ряды Лорана, а затем получим разложение . Итак, в окрестности верны разложения

Теперь можем записать разложение

Следовательно, судя по главной части ряда Лорана точка — полюс первого порядка

3 шаг. Нахождение вычетов.

6.  Вычет в точке можно находить двумя способами. Первый способ: использование главной части разложения Лорана (если такое разложение имеется). В нашем случае разложение Лорана уже выписано. Нас интересует коэффициент при , который равен . Следовательно . Второй способ вычисления вычета производится по формуле .

7.  Вычет в точке находим по формуле

8.  Вычет в точке найдем из разложения Лорана. . Отсюда

4 шаг. Исследование точки . С помощью замены надо бесконечность перевести в нуль. В результате имеем функцию , зависящую от . Надо исследовать точку . При справедливо разложение , поэтому — устранимая особая точка. Этот же вывод верен для . Вычет равен нулю.

A) , равенство 5 предела исследуемой функций,

B) , равенство 7 предела исследуемой функций,

C) , равенство бесконечности предела исследуемой функций,

D) , равенство 0 предела исследуемой функций,

E) , равенство бесконечности предела исследуемой функций.

12. Вычислить определенный интеграл при .

Решение. Цель: определенный интеграл по отрезку свести к интегралу вдоль замкнутого контура и затем применить теорему о вычетах.

Сведение к контурному интегралу.

Вычисление контурного интеграла.

A) , ,

B) , ,

C) , ,

D) , ,

E) , .

Тесты для подготовки к экзамену.

1. Интеграл от голоморфной в односвязной области функции вдоль любого замкнутого контура равен нулю

Схема доказательства.

1 шаг. Интеграл по непрерывному контуру можно приближенно заменить интегралом по специально подобранной ломанной.

2 шаг. Интеграл по ломанной в точности совпадает с суммой интегралов по некоторому набору треугольников.

3 шаг. Интеграл по достаточно большому треугольнику в точности совпадает с суммой интегралов по некоторому набору треугольников очень малых размеров.

4 шаг. Интеграл вдоль треугольника достаточно малого периметра от произвольной голоморфной функции можно приблизить интегралом по тому же треугольнику от некоторой линейной функции.

5 шаг. Интеграл от линейной функции вдоль любого треугольника равен нулю. Надо аккуратно вычислить этот интеграл.

6 шаг. Первый вывод из предыдущих шагов: исходный интеграл почти нуль. Более детальный анализ позволяет заключить, что он точно равен нулю. Теорема 6 доказана (на тройку).

Грамотное математическое оформление 1-6 шагов. Пусть замкнутый контур лежит в односвязной области . В области возьмем компакт (замкнутое и ограниченное множество) , содержащий контур . Голоморфная на функция будет непрерывной на компакте , тогда по … она на равномерно непрерывна. . На контуре выберем упорядоченный набор точек так, чтобы расстояние (по дуге) между соседними точками было меньше . Обозначим через ломанную, проходящую через точки . Оценим разность

Таким образом, 1 шаг реализован. Второй шаг на рисунке (иногда лучше рисовать, а не разговаривать). Внутри произвольно выбрана жирная точка, которая соединена со всеми вершинами. В результате разбит на треугольники. Ориентация сторон треугольников согласована с ориентацией . Тогда внутренние стороны треугольников обходятся дважды в разных направлениях. Их интегральный вклад нулевой.

Таким образом, 2 шаг реализован. Молча реализуем третий шаг (смотри рисунок).

Таким образом, периметр треугольника уменьшился в четыре раза. Никто не может возразить, если я дальше буду продолжать дробление треугольников.

Реализация четвертого шага. Поскольку подынтегральная функция — голомофная в , то в окрестности любой точки из функция хорошо приближается линейной функцией:

Пусть дан треугольник, который можно вписать в окружность радиуса . Обозначим центр описанной вокруг данного треугольника окружности через . Тогда этот …, причем можно считать . Поэтому верна оценка

Аккуратные вычисления пятого шага ( их надо привести на экзамене) дают

В силу произвольности и из того, что левая часть последнего неравенства не зависит от , получим Отсюда сразу следует . Тогда из неравенств первого шага имеем . В силу произвольности и из того, что левая часть последнего неравенства не зависит от , получим .

A) теореме Канта, треугольник лежит внутри круга ,

B) теореме Канторовича, треугольник лежит внутри круга ,

C) теореме Кангужина, треугольник лежит внутри круга ,

D) теореме Кантора, треугольник лежит внутри круга ,

E) теореме Антона, треугольник лежит внутри круга .

2. Ограниченная целая функция – постоянная функция.

Доказательство. Согласно теореме 9 в круге произвольного радиуса целую функцию можно разложить в ряд Тейлора . Оценим модуль коэффициента , если . Так как радиус произвольное число и коэффициент не зависит от , то переходя к пределу в неравенстве , получим …. Точно также оценивается . Продолжая подобные рассуждения получим, что . В результате для всех .

A) ,

B) ,

C) ,

D) ,

E) .

3. Голоморфная в открытом кольце функция разлагается в этом кольце в ряд Лорана.

Доказательство. Пусть функция голоморфна в кольце . По произвольной точке из кольца выберем числа так, чтобы . Тогда согласно ИФК имеем

Разложение в степенной ряд для дословно повторяет доказательство теоремы 9, так как в этом случае . Таким образом, Аккуратно запишем разложение в степенной ряд для .

Отсюда имеем соотношение .

A) ,

B) ,

C) ,

D) ,

E) .

Ответы к тестам на уровне определений.

1.A

2.B

3.C

4.D

5.A

6.A

7.A

8.E

9.D

10.B

11.C

12.B

13.A

14.B

15.B

16.D

17.A

18.B

19.B

20.B

21.E

22.C

23.B

24.B

25.A

26.C

27.A

28.B

29.C

30.B

31.C

32.D

33.E

34.C

35.A

36.B

37.C

38.D

39.E

40.B

41.D

42.A

43.C

44.C

45.D

46.B

47.A

48.A

49.B

50.A

51.C

52.A

53.B

54.C

55.C

56.D

57.E

58B

59.A

60.B

61.D

62.E

63.A

64.A

65.D

66.C

67.B

68.A

69.B

70.C

71.D

72.A

73.-

74.-

75.-

Ответы к тестам для подготовки к контрольным работам.

1.A

2.C

3.B

4.D

5.B

6.E

7.C

8.B

9.D

10.C

11.A

12.D

Ответы к тестам для подготовки к экзамену.

1.D

2.C

3.E

ЧАСТЬ 4

НАУЧНО-МЕТОДИЧЕСКИЕ ЗАМЕЧАНИЯ

Необходимость расширения множества действительных чисел.

Известно изречение немецкого математика Л. Кронекера (1823 – 1891) «Господь бог создал целые числа, все остальное – дело рук человеческих». На самом деле Кронекер имел в виду натуральные числа, это перевод слишком буквальный. Натуральных чисел, однако, оказывается мало. Поэтому расширяют множество натуральных чисел до множества целых чисел. Здесь возникает вопрос: каким образом определяются сложение и умножение на целых числах? Почему ? Эти вопросы не так тривиальны, как может показаться. Найти правильный ответ будет легче, если сформулировать вопрос иначе: что плохого произойдет, если считать, что ? В этом случае некоторые свойства сложения натуральных чисел не будут выполняться для целых чисел. Тут следует спросить: какие свойства сложения и умножения надо сохранять? Ведь если сохранять все свойства, введение отрицательных чисел не только излишне, но и вредно: свойство « уравнение не имеет решений», верное для натуральных чисел, становится неверным для целых! Для правильного выбора того, что нужно оставить, надо умело лавировать между слепым следованиям традициям и бесплодным новаторством.

Стандартный процесс расширения приводит к цепочке включений . Когда остановиться? Может быть, что какие-то промежуточные расширения пропущены? К примеру, пропущено — множество гипердействительных чисел. Какие расширения привлекать? Это зависит от целей, которые преследует читаемый курс. Если мы хотим, чтобы основные теоремы о непрерывных функциях на компактах выполнялись можно привлекать расширение .

Существуют разные способы расширения множеств. Добавить новые элементы и затем алгебраически замкнуть – это один способ. Другой способ – пополнение предельными элементами. Включение можно добиться первым способом, а расширение — вторым способом. Иррациональные числа представляют пределы рациональных последовательностей.

Реализация комплексных чисел.

Существуют разные способы введения комплексных чисел. Мы предпочли матричный способ, поскольку студенты-математики достаточно хорошо знакомы с матричным исчислением.

Можно комплексные числа вводить как плоские вектора. При этом трудно понять мотивацию введения умножения векторов по формуле

.

Нельзя математикам вводить мнимую единицу так: обозначим через величину, квадрат которой равен минус единице. Здесь не предъявлен объект, который обозначен через .

Топология комплексной плоскости.

Важно соблюдать равновесие между чисто математическим и естественнонаучным материалом, так как при пренебрежении первым изложение воспринимается студентами как поверхностное, а пренебрежение вторым наносит ущерб правильной оценке роли теории в науке.

Современный язык топологии позволяет с единых позиций объяснить основы как действительного, так и комплексного анализа. Однако необходима некоторая адаптация к этому языку. Студенты-математики имеют определенную практику, поэтому здесь применяется язык топологии.

Сходимость степенных рядов на комплексной плоскости.

Из-за недостатка времени нам приходится ссылаться на результаты курса математического анализа, хотя этот курс полон неожиданностей. Пусть, например, абсолютно сходящийся ряд. Верно ли, что

то есть можно ли сначала просуммировать члены ряда с четными номерами и к полученной сумме прибавить сумму членов с нечетными номерами? В обоснование этого ссылаются обычно на теорему о перестановке членов ряда из курса математического анализа. Однако требуемое утверждение не следует из этой теоремы, так как в результате перестановки должен получиться снова ряд, а не сумма двух рядов вида . Надо приспособить доказательство теоремы о перестановке членов ряда к указанному случаю, однако в курсе математического анализа этого не делается. Трудно поколебать сложившуюся традицию!

Наша цель: как можно быстро познакомить студента с теми методами, которые специфичны для ТФКП, а не критиковать практику преподавания курса математического анализа.

: Голоморфные функции.

Здесь изучаются голоморфные в открытой области функций. Иногда вместо «голоморфные функций» говорят «регулярные функций». Мы понятие «аналитические функций» сохранили для полных аналитических функций, которые могут быть многозначными.

Поскольку изучаются голоморфные в открытой области функций, то в лекциях не исследуются свойства интегралов типа Коши, граничные свойства функций и так далее.

Интегрирование функции комплексной переменной.

При введении понятия интеграла комплекснозначной функции вдоль контура на плоскости долг преподавателя вновь и вновь пропагандировать ту старую истину, которую еще Петр пытался ( безуспешно ) внушить русским купцам : что торговать надо честно, без обмана, так как в конечном счете это для самих же себя выгоднее.

Мы попытались студенту-математику самому выбрать одно из пяти разных определений интеграла функции вдоль контура. Студентам инженерных специальностей – это роскошь!

Интегральная теорема Коши.

Интегральная теорема Коши, на самом деле, есть далеко идущее следствие того, что интеграл

принимает только целые значения, и того, что интеграл – непрерывно зависит от контура интегрирования. То есть если контур поменять на другой (в некотором смысле близкий к ), то в силу непрерывности интеграл сохраняет прежнее значение. Этот факт остается справедливым для многочленов, так как многочлен – сумма целых степеней . Дальше указанный факт можно распространить на степенные ряды. Поэтому, в принципе, справедливость интегральной теоремы Коши никем не оспаривается и основное внимание следует сосредоточить на ее следствиях. Подчеркивание того обстоятельства, что интегральная теорема Коши прежде всего очевидна для многочленов, чрезвычайно облегчает студентам понимание того, чего от них хотят.

Интегральная формула Коши.

Автор этого курса лекций старался познакомить с калейдоскопом фундаментальных идей математики студентов-математиков с минимальными познаниями. Вместо обычного в математических книгах принципа наибольшей общности автор старался придерживаться принципа минимальной общности, согласно которому каждая идея должна быть вначале ясно понята в простейшей ситуации, и только затем развитый метод переноситься на более сложные случаи.

Обратите особое внимание на обсуждение интегральной формулы Коши. Каждый лектор, следуя собственным научным вкусам, может продолжить обсуждение этой теоремы.

Локальное разложение голоморфных функций в степенные ряды.

Собственно математическая часть курса построена по принципу выведения наибольшего числа следствий из наименьшего числа общих теорем. Интегральная теорема Коши – это общий факт, из которого следует много разных по характеру и значимости следствий. Здесь обсуждается связь между дифференцируемостью и представлением функции в виде степенного ряда. Из этой связи вытекают многие свойства голоморфных функций.

В курсе математического анализа радиус сходимости ряда Тейлора определяется по формуле Коши-Адамара. Вне круга сходимости ряд Тейлора перестает сходиться. Причину этого нельзя понять, оставаясь в действительной области. Выход в комплексную область сразу разъясняет явление: на окружности, являющейся границей круга сходимости, лежит точка, в которой значение ряда обращается в бесконечность. Таким образом, радиус сходимости степенного ряда равен расстоянию от центра до ближайшей особой точки. К этому кругу утверждений можно отнести известные теоремы Прингсхейма.

Целые функций. Теорема Лиувилля.

Одним из самых эффективных с точки зрения приложений разделов ТФКП является теория целых функций. Важную роль играют теорема Фрагмена-Линделефа, теорема Лиувилля и принцип максимума. Последние два утверждения приведены в лекциях. Теорема Фрагмена-Линделефа утверждает, что ограниченная на сторонах угла целая функция – ограничена также внутри угла, если раствор угла согласован с порядком роста целой функций. К сожалению, в данном курсе нам не удалось продемонстрировать возможности теории целых функций.

Понятие аналитического продолжения.

Понятие продолжения в математике возникает в отдаленных друг от друга ее разделах. К примеру, при вещественных сумма ряда равна . Если вместо подставить комплексное число , то получим комплексные значения экспоненты. Если вместо подставить матрицу , то получим матричное значение экспоненты. Если вместо подставить оператор , то получим операторное значение экспоненты. Таким образом, имеем продолжения экспоненты на множества элементов различной природы. Спрашивается: на какое максимальное множество той или иной природы можно продолжать? Спектральная теория операторов отвечает на этот вопрос, когда продолжаемое множество состоит из операторов. Здесь продолжаемое множество представляет комплексные числа.

Часто продолжения должны обладать наперед заданными свойствами. В данном случае речь идет об аналитичности. Аналитичность можно сохранить, если продолжать по непрерывности, по Эйлеру, по Коши, по Вейерштрассе, по Риману и так далее. Мы здесь остановились только на некоторых методах продолжения.

Понятие римановой поверхности.

Оказывается, что область определения аналитической функции не всегда представляет собой часть плоскости. Иногда — это многолистное множество, где листы склеены особым образом. То есть области определения функций могут быть сложными множествами. К счастью, в данном разделе науки — это гладкие многообразия или поверхности Римана. Таким образом, возникает проблема изучения объектов, определенных на многообразиях. В университетских курсах математические объекты чаще всего определены на отрезках, прямых, прямоугольниках, плоских фигурах и так далее. Однако на практике встречаются объекты, определенные на более сложных множествах. К примеру, процесс таяния снежинки приводит к задаче теплопроводности на фрактале. Современный язык математики еще не адаптирован к решению таких проблем. Подобные области математики, особенно богатые связями со многими явлениями природы, все больше и больше привлекают к себе внимание. По-видимому, наступило время, когда изучающие математику должны знакомиться с анализом на многообразиях, на фракталах и так далее.

Принцип аргумента.

Многие утверждения комплексного анализа основаны на принципе линеаризации. То есть исследуемый объект приближается линейным агрегатом, свойства которого наследуются исследуемым объектом. Принцип аргумента – это инструмент нелинейного (нелокального) анализа. Обобщением принципа аргумента в глобальном анализе является понятие степени вращения гладкого отображения, которое не меняется при гомотопиях. Степень отображения выражается через интегралы от дифференциальных форм и принимает только целые значения. Этот факт в нашем случае означает, что величина — целое число.

Теорема Руше.

Теорема Руше – непосредственное следствие принципа аргумента. Прием, с помощью которого доказана теорема Руше, называют « дама с собачкой». Вот, что пишет по этому поводу А. Тоом в «Кванте» / 1990. № 2/ : «Давным давно, когда я был студентом мехмата МГУ, один из старших друзей спросил меня: «Знаешь «Даму с собачкой»?» — «Ну конечно», — ответил я, имея в виду рассказ А. П.Чехова. – «Нет, — сказал собеседник, — я про другое. Есть такое доказательство основной теоремы алгебры». И тут же рассказал мне очень красивое (хотя и нестрогое) рассуждение, которое в итоге оказалось одним из самых ярких моих впечатлений за все годы учебы. Однако, несмотря на свою наглядность, доказательство это, давно уже вошедшее в «математический фольклор», по-моему, так до сих пор и не записано в своем наиболее доступном виде».

[1] На самом деле возникнет расширение, которое не совпадает с. Какое надо взять уравнение вместо, чтобы расширение совпало с множеством действительных чисел? Не спасает положение даже уравнение с многочленом, у которого коэффициенты произвольные рациональные числа.

[2] А может надо было исходить из другого неразрешимого в уравнения? К примеру, .

[3] Необходимые определения приведены в последующих лекциях.

[4] В других источниках называют условиями Даламбера-Эйлера.

[5] На самом деле,

[6] Дополнительный вопрос для студентов кафедры математического и компьютерного моделирования.

[7] Считаем, что кривые задаются непрерывно дифференцируемыми функциями, то есть кривые Пеано, фракталы исключаются.

[8] Г. Харди Двенадцать лекций о Рамануджане. Москва. ИКИ. 2002.336 с.

[9] Рамануджан получил интересные формулы, некоторые из которых современные математики доказывают с помощью теоремы Коши.

[10] Lioville Joseph (24.03.1809 – 8.09.1882) – французский математик

[11] Левин Б. Я. Распределение корней целых функций М. – Л. Гостехиздат, 1956 632 с.

[12] Получена Дж. Морерой в 1886 году. Фамилия склоняется. Смотри Мат. Энц. Словарь.

[13] Где-то в определений голоморфности присутствует требование однозначности отображения. В случае аналитичности такого требования нет, требуется возможность локального разложения в степенной ряд.

[14] Это следует из того, что таковой является правильная часть разложения Лорана.

[28] Прочитайте исторический очерк ТФКП из книги А. И.Маркушевич, Л. А.Маркушевич Введение в теорию аналитических функций. М.:Просвещение. 1977. 320с.

[29] Так как функция непостоянна

[17] Смотри предисловие. П. Монтель Нормальные семейства аналитических функций.- М.-Л.,1936

[18] Обход контура против часовой стрелки.

[19] Потому что интеграл зависит также от числителя.

[20] Спрошу на экзамене.

[21] Для тех кто желает получить высший бал на экзамене.

Наташа

Автор

Наташа — контент-маркетолог и блогер, но все это не мешает ей оставаться адекватным человеком. Верит во все цвета радуги и не верит в теорию всемирного заговора. Увлекается «нефрохиромантией» и тайно мечтает воссоздать дома Александрийскую библиотеку.

Распродажа дипломных

 Скидка 30% по промокоду Diplom2020